Sei sulla pagina 1di 48

Term 5 Challenges

Spring 2011
Questions from the exam that I had trouble with
__=question that I still do not know the correct answer to even after going back to the book and the
notes, or maybe the question was ambiguous/unfair. Let me know if you ever find the answer to the
question because we're not allowed to discuss exam questions with the professors after taking the
exam. Darned sequestration policy. I've compiled this because of that, so be smart if sharing with
others and make sure the professors do not get a hold of this, and especially that my name does not get
attached to it please. Enjoy!

BSFCR & Pathophysiology


MEQ 1- Infections
A 47 year old with an 18-pack-year history presents with fever, hemoptysis, and dyspnea. He has a
history of COPD, and sputum culture reveals Streptococcus pneumoniae.
1. What lab test would be done to check for the presence of capsule in this microorganism?
Quellung reaction
2. What is the most common normal flora of the oropharynx?
viridans Streptococcus
3. If the microorganism had been Mycobacterium tuberculosis, what special culture media
would be needed for it to grow?
Löwenstein-Jensen agar

A wildlife photo journalist presents with a 5-day fever. He had been hiking for 2 days in the forest
before his present symptoms. A tick was found on his body. He is diagnosed with Lyme disease.
4. What is the pathogenetic organism responsible for this disease?
Borrelia burgdorferi
5. Which zoonosis can be acquired by ingesting unsterilized milk or dairy products?
Brucellosis

A 6 year old boy presents with cough, pharyngitis, and conjunctivitis. Investigation reveals an
infection by adenovirus.
6. This virus contains what type of nucleic acid?
DNA
7. Name a single-stranded DNA virus that can infect humans.
Parvovirus, Circovirus

A teacher presents with a skin rash consisting of pustules and vesicles. She is diagnosed with a
superficial fungal infection.
8. Give a general name of one type of fungus that could cause this infection.
Trichophyton, Epidermophyton, Microsporum
9. Name a method that can be used to visualize the fungi.
KOH prep

An intravenous drug user is diagnosed with AIDS.


10. What number is important in determining the progression of disease?
CD4+ count <200
MEQ 2- Cardiology
An obese middle-aged male smoker presents to the ER with chest pain radiating to the neck and arm.
Cardiac biomarkers were found to be elevated, and he was diagnosed with an MI. He has an end-
diastolic volume of 140 mL, an end-systolic volume of 105 mL, and a heart rate of 100 beats/min. He
is a right coronary dominant person.
1. What is his cardiac output?
SV = EDV-ESV = 140-105 = 35 mL
CO = SV x HR = 35 mL/beat x 100 beats/min = 3500 mL/min
2. If the MI caused an infarct at the apex, which coronary artery was occluded?
Left anterior descending artery.
3. Pain from the heart travels along splanchnic nerves to which levels of thoracic spinal
nerves?
T1-T5

A 5 year old boy was brought in with dyspnea and dysphagia. Upon examination he has a pansystolic
murmur, and a transesophageal echocardiogram reveals he has an atrial septal defect.
4. The most common type of atrial septal defect results from a failure of closure of which
foramen?
Ostium secundum
5. His dysphagia results from the compression of the esophagus by which chamber of the
heart on the posterior surface?
Left atrium

An elderly man with mitral stenosis due to rheumatic etiology comes in with bilateral rales, bilateral
pedal edema, elevated jugular venous pulse pressure, and a loud S1 sound.
6. Where is the best place to listen for the mitral valve irregularity?
Left 5th intercostal space midclavicular line
7. Which specific pressure is altered, leading to the edema?
Capillary hydrostatic pressure

The most significant portion of Tetralogy of Fallot to determine prognosis is the amount of pulmonary
stenosis.
8. Tetralogy of Fallot results from the abnormal migration of which cells?
Neural crest cells

A man is found to have yellow deposits around the eyelids, and has a high blood cholesterol. He has
several family members who are diagnosed with hyperlipoproteinemia IIa, and he is diagnosed with it
too.
9. Hyperproteinemia IIa results from a deficiency of hepatic receptors to what?
LDL
10. Statins are given to treat this disease by inhibiting what enzyme?
HMG-CoA reductase

MEQ 3- Respiratory
I got a 9/10 on this one for some reason, but I don't know what is incorrect. And I will never be able to
find out because of sequestration policy, so I don't know.

A 47 year old man presents with pulmonary tuberculosis and is found to have right pleural effusion.
His alveolar ventilation is found to be lower than normal.
1. What specific lung volume is decreased, bringing about the decrease in alveolar
ventilation?
Tidal volume
2. Which border of the rib should a thoracocentesis be done in order to prevent disturbance
of the neurovascular bundle?
Upper
3. Name two factors that play a role in the activity of the peripheral chemoreceptors.
ĻPO2, ĹPCO2, Ĺ[H+]

A 53 year old man with 20 pack-year history of smoking comes in with a barrel-shaped chest, is using
accessory respiratory muscles, and is dyspneic. He is diagnosed with emphysema.
4. Which lung volume (not capacity) is increased in the patient?
Residual volume
5. What factor released by neutrophils and macrophages plays a role in the pathogenesis of
the disease?
Elastase
6. With the etiology of smoking, which type of emphysema do you expect the patient to have?
Centriacinar

A 44 year old woman comes in with dyspnea and chest pain. She has been diagnosed with DVT, and a
CT scan shows the occurrence of pulmonary thromboembolism in her right main pulmonary artery.
7. What alteration in ventilation/perfusion matching has happened here?
Ĺdead space
8. What is the epithelial lining of the trachea?
Ciliated pseudostratified columnar with Goblet cells
9. Why would an aspirated object be more likely to end up on the right bronchus?
Wider and steeper

A young infant boy has been having trouble feeding and recurrent infections. CT imaging reveals that
there was a failure of the respiratory and digestive tract to fully divide and thus an aberrant tract
connects the two.
10. What type of anomaly is this called?
Tracheo-esophageal fistula

MEQ 4- Immunology
I did not receive a grade for this quiz, so that means it is my dropped grade and thus my lowest quiz
grade (<9/10), but I do not know what I got wrong and won't be able to find out either because of
sequestration policy.
1. A young boy gets scraped while playing on the school ground. Microorganisms that enter
the wound are destroyed by macrophages in the lymphatic system. This is an example of
what kind of immunity?
Innate immunity
2. What are the antigen-presenting cells in the skin called?
Langerhans cells
3. CD4+ T helper cells recognize antigens processed and presented by which type of antigen-
presenting molecule?
MHC Class II
4. What is the name of the membrane-associated lymphoid tissue found in follicles within the
lamina propria of the ileum?
Peyer's patches
5. Which is the primary immunoglobulin secreted in bodily secretions such as saliva?
IgA
6. Which immunoglobulin has the highest levels in circulation?
IgG
7. Which immunoglobulin is expressed on immature B-cells?
IgD IgM
8. A 4 year old boy with recurrent infections presents to the ER with another respiratory
infection. Physical examination reveals bilateral rales. Genetic testing reveals the boy has
a btk kinase mutation. This results in what blood cell developmental defect?
Pro-B cell to Pre-B cell development
9. A 37 year old woman presents with malar rash, joint pains, and glomerulonephritis. She
is diagnosed with systemic lupus erythematosus. Antigen-antibody complexes deposit in
the skin, joints, and glomerulus to cause the damage in this disease. This is an example of
what kind of hypersensitivity reaction?
Type III
10. A 27 year old man received a kidney transplant 2 years ago but now suffers from
hematuria and is found to have high blood creatinine levels. He is found to have chronic
rejection of the kidney transplant. What morphology would you expect to see on a biopy
of the kidney?
Arteriosclerosis Luminal obliteration and
interstitial fibrosis
Exam 1
Which part of a Gram negative bacteria's cell surface contains the component responsible for
inducing septic shock?
The Lipid A causes septic shock, but where is this located- in the capsule, cytoplasmic membrane, outer
membrane, periplasmic space, or peptidoglycan? The answer is outer membrane.

A 38 year old lady presents with an eruption of vesicles and blisters in a characteristic pattern
following her T12 dermatome. What nucleic acid class does this virus belong to and is it
enveloped?
She has HSV-3 Varicella-zoster, which is an envoloped dsDNA virus.

An AIDS patient has a history of dyspnea, cough, and bilateral rales heard upon auscultation. A
CD4 count reveals <50. What microorganism is this patient at risk of being infected by?
I challenged this question because First Aid for the USMLE 2010 states on pg. 175 that could be
Mycobacterium avium complex or cryptococcal meningoencephalitis, which both were answer choices.
But according to the lecture notes, Mycobacterium avium complex is at CD4 levels <50, whereas
Cryptococcus neoformans is at CD4 levels <200.

What is the progression of CD4 levels in a patient from onset of HIV infection to the development
of AIDS?
Initial drop in CD4 levels, followed by a rebound spike, and then gradual loss until immune
dysfunction.

An 8 year old child is brought in by his parents because he has blisters and vesicles over his skin
with hyperemia around them. Some have burst and crusted over. He is diagnosed with a bullous
impetigo. What is the most likely organism?
Staph aureus. I got this wrong because I vaguely remembered that non-bullous impetigo can be caused
by either Staph aureus or Strep pyogenes,, but still even for that, Staph aureus is the predominant
organism.

A 3 year old infant begins flailing its limbs before continuing into a state of stupor. A bulging
fontanelle is noted, and CSF reveals Gram negative cocci. What is a characteristic of this
bacteria in lab tests?
I am guessing this is Neisseria meningitidis, though I expect it to infect older patients, but it is the only
Gram negative cocci I know. N. meningitidis can ferment maltose, whereas N. gonorrhoeae cannot,
which is how the two are differentiated from one another.

A 27 year old woman goes to the STI clinic because she has been having vaginal discharge for the
past few days. A swab is taken and cultured, resulting in Neisseria gonorrhoeae. Where is the
best place to do a swab?
I don't know where we learned this, but I chose endocervical canal because that is where mucus plug
prevents pathogens from entering uterus, and I think it is correct.

A man with a 10 pack-year history of smoking and history of diabetes is rushed to the hospital
after an MI. On his 5th day of recovery, he begins to feel pain in his chest again that radiates to
his shoulder. A blood sample is taken 1 hour after initiation of this new pain to check if he is
having a recurrent MI. Which cardiac enzyme can be checked for in order to diagnose a 2nd MI?
Myoglobin. This takes 1-2 hours to elevate, whereas the other markers CK-MB and troponins would
have taken 4-6 hours to elevate. LDH takes 10-12 hours.

A 56 year old man is rushed to the hospital with pain in his chest and radiating down his arms.
He is diaphoretic and dyspneic. EKG readings show ST elevations in Leads II, III, and aVF.
Which coronary artery is most likely blocked?
These findings indicate an inferior wall MI, which I believe to be supplied by the posterior descending
branch of right coronary artery, not the marginal branch of the RCA nor any other coronary artery.

A woman suffering from pulmonary edema coughs up blood-tinged sputum. Which nerves are
responsible for the cough reflex?
I got this wrong. I said non-myelinated C fibers of afferent pathway of vagus nerve, but these innervate
receptors in the interstitium and interalveolar spaces. The myelinated fibers of afferent pathway of
vagus nerve innervate the larynx, large airways, and proximal bronchi, and are thus responsible for the
cough reflex.

A woman presenting with progressive dyspnea and non-productive cough comes in to have her
condition checked out. Pulmonary function tests show her total lung capacity and forced vital
capacity are reduced. She is found to have an elevated erythrocyte sedimentation rate,
hypergammaglobulinemia, and anti-nuclear antibodies. What is her likely diagnosis?
Idiopathic pulmonary fibrosis, which is also known as usual interstitial pneumonia. Interestingly
enough, you may also find rheumatoid factor in patients with such disease.

A 37 year old lady presents because she is amennorheic, obese, has pronounced hirsutism and
profuse acne on the face. Ultrasound reveals multiple cysts in her ovaries bilaterally. What
would you expect her hormone levels to be like?
ĻFSH, ĹLH, ĹAndrogens. I got this wrong, I switched FSH and LH around.

A G1P0 in her 36th week of pregnancy has headaches and is found to be hypertensive. She
progesses to get seizures and is in critical condition. What other clinical finding may you find in
this patient?
I challenged this question because the HELLP syndrome includes hemolysis, elevated liver function
tests, and low platelets. So I would expect to find low RBC count, as well as low platelet aggregation,
both of which were answer choices.

A patient with menstrual abnormalities has her hormone levels checked, and no FSH nor LH is
detected. Graph shows that no progesterone is detected either, but estrogen can be detected
throughout the menstrual cycle. What is the pathology could present with these findings?
I don't know. Either estrogen-secreting ovarian tumor or hypothalamic-pituitary dysfunction.

MEQ 5- Hematology
A man presents with pallor, fatigue, and hepatomegaly. He underwent total gastrectomy due to
carcinoma of the stomach a few years ago. Peripheral blood smear shows megaloblastic anemia.
1. What factor is deficient that results in his inability to absorb Vitamin B12?
Intrinsic factor
2. What metabolite of Vitamin B12 can you check to differentiate from folic acid
deficiency?
Methylmalonic acid
3. What would be the characteristic morphology of neutrophils in peripheral blood smear?
Hypersegmentation

A confused man comes in and is found to have polycythemia vera.


4. What mutation results in this disorder?
JAK-2. I said JAK-STAT, so I don't know if they will accept, but it is actually a JAK-2
mutation that causes constitutive activation of the JAK-STAT pathway.
5. What would his erythropoietin levels be (low/normal/high)?
Low
6. He has an increased likelihood to develop hepatic vein thrombosis. What is this
complication called?
Budd-Chiari Syndrome

A 26 year old woman goes for consulting purposes because her 6 year old brother has sickle cell
disease, which is an autosomal recessive disorder.
7. What are the chances (percentage) that she is a carrier of the sickle cell gene?
67%
8. What are the components of Hemoglobin A2?
Alpha, delta

A 2 year old boy suffers from bleeding at the mucosal membranes and the joints. He has a 6 year old
sister who is unaffected, but an 8 year old brother who has similar problems. He is found to have
Hemophilia A.
9. What is the mode of inheritance of this disorder?
X-linked recessive
10. How would the PT and PTT be affected?
Normal; increased

MEQ 6- GIT
A lady with complaint of dysphagia comes in and is found to have achalasia.
1. The patient's problem is due to a deficiency of which nerve plexus?
Auerbach's = myenteric
2. The patient would be found to have an increased pressure in which part of the esophagus?
Lower esophageal sphincter
3. Secondary peristalsis is initiated by what?
Reflex to residual contents in the esophagus, distension? I thought this was a bit ambiguous
what exactly they were looking for here.

A man comes in with complaint of abdominal pain and vomiting of blood. He is found to have peptic
ulcers.
4. If the peptic ulcer were to perforate the stomach wall at the lesser curvature close to the
pylorus, which artery would be eroded?
I said gastric artery because the left and right gastric artery connect at the lesser curvature, but
the right gastric artery is closer to the pylorus if they want to be that specific.
5. Which cells in the stomach secrete pepsinogen?
Chief cells
6. Which enzyme helps increase the production of hydrogen ions to be secreted by the gastric
mucosa?
Carbonic anhydrase

A patient with past history of abdominal problems presents with an acute attack of abdominal pain and
diarrhea. An endoscopy reveals transmural inflammation in the ileum and colon with skip lesions and
sparing of the rectum.
7. What is your diagnosis?
Crohn's disease
8. Which enzyme in the small intestine is responsible for activating pancreatic enzymes?
Enterokinase/enteropeptidase

A man has lower abdominal pain is found to have a herniation lateral to the inferior epigastric artery
that protrudes when he coughs.
9. What kind of hernia is this?
Indirect inguinal
10. What is the medial border of Hesselbach's triangle?
Lateral border of rectus abdominis

MEQ 7- Hepatobiliary & Male Genital


A 73 year old long-time sufferer of cirrhosis presents to the emergency department in a confused state
and vomiting large amounts of blood. He is unable to perform simple commands. The attending
physician puts in an order for liver transplant.
1. The hemetemesis is a result of rupture of the porto-caval anastamosis between left gastric
vein and what other vein?
Esophageal vein
2. The abnormal neurological findings are a result of increased serum levels of what?
Ammonia
3. For the liver transplant, the hepatoduodenal ligament will be lysed, cutting off flow
through the hepatic artery, common bile duct, and what other vessel?
Portal vein
4. Which blood vessel is found in Zone III of the liver lobule?
Central vein
A 1 year old boy with chromosomal makeup 46XY is found to be missing a penis, but has a small
vagina that ends as a pouch. Ultrasound shows the presence of vas deferens and epididymis, but not of
uterus or fallopian tubes. Normal testicular gonads are found in the inguinal area.
5. What substance is deficient in the patient, which explains these findings?
5Į-reductase
6. Which embryological duct gives rise to the male internal genitalia?
Mesonephric/Wolffian duct

A 63 year old man has incontinence and trouble maintaining an erection. Examination reveals that he
is suffering from prostatic carcinoma.
7. Which nerve is affected, causing the problem of incontinence and impotence?
Pelvic splanchnic nerves
8. Which zone of the prostate usually gives rise to the prostatic carcinoma?
Peripheral zone

A 23 year old man is found to have low sperm count. His testosterone levels in the seminiferous fluid
are much lower than in the serum.
9. What substance is deficient in his seminiferous fluid?
Androgen-binding protein
10. Which cells found in the testes usually secretes this substance?
Sertoli cells

MEQ 8- Renal
A man comes in with severe, colicky pain in the right flank and hematuria. Diagnostic imaging reveals
that he is suffering from a kidney stone.
1. Where is the lowest constriction of the ureter where a stone is likely to become lodged?
Ureterovesical junction
2. Which spinal nerve segments are responsible for the pain that is felt?
T11-L2
3. The failure of development of which embryological structure leads to renal agenesis?
Metanephric diverticulum

A male patient with 20 year history of diabetes is found to have a urine creatinine concentration of 120
mg/day, urine flow rate of 0.5 L/day, serum creatinine of 1 mg/dL, and serum glucose 3 mg/dL.
Assume glomerular filtration rate is equal to creatinine clearance.
4. Calculate the patient’’s creatinine clearance.
UCr*V/PCr = 120 mg/dL*0.5 dL/min / 1 mg/dL = 60 dL/min
5. Calculate the filtered load of glucose.
Filtered load = GFR*Pglucose = 60 dL/min*3 mg/dL = 180 mg/min

A child is brought into the emergency room because of sudden onset of proteinuria with resultant
periorbital edema.
6. Which specific Starling force value has changed, causing the edema?
Decreased capillary oncotic pressure
7. The filtration coefficient relates the net filtration pressure to GFR. It is based off two
values, the permeability of the filtration membrane and the _______________?
Surface area
8. Where in the nephron is most of the sodium reabsorbed?
Proximal convoluted tubule
A patient with history of proteinuria and edema, has a kidney biopsy taken, and it is shown that there
are subepithelial deposits with diffuse glomerular basement membrane thickening, creating a spike and
dome appearance.
9. What is the diagnosis?
Membranous nephritis
10. What would the biopsy look like under immunofluorescence?
Granular deposits of Ig and C3 in subepithelial space

Exam 2
A young boy is brought in by his mother because he has been failing to thrive. On physical exam
the child has no energy and is found to have prominent facial bones and hepatomegaly.
Hemoglobin electrophoresis show high levels of hemoglobin A2 and hemoglobin F, and there are
low levels of ȕ globin chain. The child has ȕ-thalassemia. What would you expect to find on a
peripheral blood smear?
Microcytic hypochromic anemia with anisopoikylocytosis. Wasn’’t sure if it had anisopoikylocytosis or
or not, but yes it does.

A woman comes in because she has been having a tingling feeling in her arms and legs, and she
has diminished position sense and sensation to vibration. She became a vegan 2 years ago.
Peripheral blood smear shows macrocytes and hypersegmented neutrophils. There is also
increased bilirubin levels and increased lactate dehydrogenase. What causes these increased
levels?
Intramedullary hemolysis, not hyperactive spleen.

A 63 year old man presents with bilateral cervical lymphadenopathy. If this were Hodgkin’’s
lymphoma, what would the lymph node biopsy look like under microscope?
Mirror-image nuclei (““owl’’s eyes””) tumor cells in a background of histiocytes, lymphocytes, plasma
cells, eosinophils.

A man is found to have cancer of the blood. Morphology shows the presence of centrocytes. You
would expect to find a mutation involving which gene in this patient?
A centrocyte is a B cell with a cleaved nucleus, which is found in follicular lymphoma, so you would
expect to find increased bcl-2 expression.

Clinical vignette of hematopoietic cancer. Neoplastic cells have CD10, CD19, and Tdt. What
kind of blood cancer is this?
Immature B cell neoplasm. Not mature because still has Tdt.

A 30 year old woman presents with increased susceptibility to infections, sluggishness, and
ecchymoses all over the body. Reticulocyte counts are low. What is the most likely cause of her
symptoms?
I challenged this question because I know that there is definitely something wrong with the bone
marrow as there is pancytopenia with low reticulocyte counts. So my first thought is aplastic anemia,
so I chose the answer choice of hypocellular bone marrow with increased amount of fat. But I thought
the other answer choices were acceptable etiologies of the pancytopenia as well: invasion of bone
marrow by leukemia, primary myelofibrosis. I only chose aplastic anemia because it seems like the
most common cause, and majority of the times idiopathic. Also her young age would make her less
likely to develop cancer, but you never know……
A male patient presents with complaint of prolonged lower back pain and tiredness. X-ray
reveals lytic bone lesion. Peripheral blood smear shows rouleaux formation of red blood cells.
Bone marrow biopsy reveals neoplastic cells with cartwheel-like nuclei. What do these neoplastic
cells secrete?
I wasn’’t sure between IgG with kappa only or IgG with kappa and lambda. I chose IgG with kappa
only because I think the neoplastic cells are monoclonal and would have only one type of light chain.

The upper esophageal sphincter is formed by which muscle?


Cricopharyngeus muscle. Got this wrong- chose superior pharyngeal constrictor, but it is the inferior
pharyngeal constrictor (a.k.a. cricopharyngeus).

A man presents with severe abdominal pain and coffee-grounds vomitus. He is found to have
perforation of an ulcer at the posterior wall of the duodenal bulb. Which artery was perforated
in this circumstance, causing the bleeding?
Gastroduodenal artery. Remember that the celiac trunk gives rise to the common hepatic artery, left
gastric artery, and splenic artery. Common hepatic artery gives rise to right gastric which anastamoses
with the left gastric. Also off of common hepatic is gastroduodenal, which gives rise to right gastro-
omental(a.k.a. gastro-epiploic), which anastamoses with left gastro-omental which comes off the
splenic.

A patient with liver failure is found to have a mutation of the ceruloplasmin gene ATP7B. What
findings would you expect for serum copper levels, serum ceruloplasmin levels, and urinary
copper levels?
Ĺserum copper levels, Ļserum ceruloplasmin levels, Ĺurinary copper levels

An elderly lady with history of abdominal discomfort, now recent blood-stained stools. There is
no history of pain, diarrhea, or trauma. What is the likely cause?
Diverticulosis, hemorrhoid, or angiodysplasia? I chose angiodysplasia, but don’’t think this is correct
because it is found more in ileum, cecum, and ascending colon. Diverticulosis in sigmoid colon, and
hemorrhoid in anus, and both can cause painless bleeding, so I do not know the correct answer.

Don’’t remember the question, but it required knowledge of how to differentiate hyperplastic,
juvenile, Peutz-Jeghers, inflammatory, and familial adenomatous polyps.
Hyperplastic are found in rectosigmoid and non-neoplastic. Juvenile are primarily made up of lamina
propria, so this was not the answer choice. Peutz-Jegher polyps have a smooth muscle core, and may
be found with hyperpigmented parts of body. Inflammatory polyps found in ulcerative colitis due to
regenerating mucosa. I chose familial adenomatous polyp for some reason indicated by the clinical
vignette.

Part of a two-part question in which 30 year old lady has had chronic diarrhea and has never left
the country before. Biopsy shows blunting of the villi and inflammation of the lamina propria.
For the first question, you have to figure out this lady has celiac disease and that you would find
anti-endomyseal IgA. Second question asked which kind of nutrient deficiency anemia would
you find?
Iron deficiency anemia

An immobile patient begins to develop an ulcer on the buttocks. The ulcer has full-thickness skin
loss and damage to the subcutaneous tissue but no penetration of the underlying fascia and no
damage to muscle, bone, or supporting structures. The ulcer crater contains necrotic tissue and
exudate. What stage is this pressure ulcer?
Stage III

Remember that Prehn test is negative (scrotal elevation does not decrease pain) in testicular
torsion and positive in acute epididymitis (elevation relieves pain).

A couple present to the clinic due to infertility issues. The female was found to be fertile. The
male has long legs, slight gynecomastia, sparse facial hair, and karyotype reveals 47XXY. What
is the reason for the infertility?
Seminiferous tubule dysgenesis. I wasn’’t sure with this one because I thought loss of Sertoli cell
function could have been an acceptable answer, but I guess in Klinefelter syndrome, they never
developed to be able to have a loss of function.

Sertoli cells secrete inhibin which have a negative feedback mechanism on the release of what
substance?
I challenged this question because the notes state that inhibin and activin both have effect on the release
of LH and FSH, but apparently this is wrong and inhibin only downregulates FSH whereas activin only
upregulates FSH.

A male patient with infertility problems gets tested and is found to have low testosterone levels,
low LH levels, and high FSH levels. What may have caused these lab findings?
I was confused because of the LH-FSH mismatch. Damage to Leydig cells would cause low
testosterone. This would cause an increase in LH in response, but if the anterior pituitary were
damaged too, then there would be low LH levels, but then one would expect to find low FSH levels
too. I chose damage to Leydig cells and lactotrophs, but think this is wrong because lactotrophs secrete
prolactin. A more plausible answer would be damage to Leydig cells and gonadotrophs, but still I am
confused about the cause for the LH-FSH mismatch.

A patient is receiving a kidney transplant. Image is shown of posterior surgical approach to


obtain kidney, and various nerves in the vicinity. Have to identify one of the labeled nerves.
Used memory of mnemonic from Anatomy to solve this one. Lumbar plexus- (So I’’m In Grenada
Living Free Of Luxury = 12th Subcostal, Iliohypogastric, Ilioinguinal, Genitofemoral, Lateral femoral
cutaneous, femoral, obturator, lumbosacral trunk). Labeled nerve was third one down, so must have
been ilioinguinal which is at risk for injury during renal transplant.

A patient with progressive end-stage renal disease has imaging done, which shows bilateral
shrunken kidneys. What electrolyte imbalance would you expect in this patient?
Hyperkalemia

The kidney aids in maintenance of blood pH by which performing what function?


One of these was the correct answer: bicarbonate reclamation in proximal tubule cell; bicarbonate
regeneration in distal tubule cell.

A male patient with uncontrolled hypertension is found to have fibrinoid necrosis of the renal
artery. What is the kidney’’s first response to protect itself from the high blood pressure?
Not sure but I put arteriolar narrowing.

A lady with history of thromboembolic events has a renal infarct. How does this cause a decrease
in the glomerular filtration rate?
I wanted to challenge this but ran out of time. Does the renal infarct decrease renal perfusion, decrease
filtration coefficient of the net filtration pressure, or decrease the surface area of infiltration? I would
believe that any of these occur in a renal infarct. Don’’t know what the correct answer is.

An athlete loses a significant amount of sweat after intense exercise. Later, he drinks two liters of
water. This decreases the osmolality of his plasma. What else would you expect to find?
I have no idea how to answer this. Answer choices were about changes in tonicity of intracellular and
extracellular fluid, or increased intracellular volume. I do not know how these are affected by sweating
and drinking.

A 23 year old male is brought into the emergency department unconscious after a night of
partying. A bottle of sweet-tasting liquid is found in his possession. A list of plasma values are
given. He is found to have oxalate crystals in his kidney as well as epithelial casts in the urine.
This substance causes the most damage to which part of the kidney?
The patient has ethylene glycol poisoning, which damages primarily the proximal convoluted tubule.

A man suffers from head trauma and after the incident, he presents with polyuria and
hypoosmotic urine. What can you say about his sodium content?
You must identify this case as diabetes insipidus and thus dysfunctional retention of water. There is
hypernatremia but due to pure water loss, and so body total sodium content will remain unchanged.

A male patient has low urine output after surgery, but has since drunk a lot of water. Normal
urine osmolality is usually between 50-1400 mOsm/kg. Normal urine sodium is 25-250 mmol/L.
Comment on what you would expect to see in the osmolality and sodium concentration of the
urine.
I have no idea.

A week after having an upper respiratory tract infection, a young boy presents with cola-colored
urine and periorbital edema. What would you expect to find on kidney biopsy?
I was looking for the answer of hypercellular with neutrophilic infiltrate, but this answer was not
available. But there was subepithelial immune complex humps, which is the correct answer.

MEQ 9- Psychiatry, Behavioral Sciences, Epidemiology, Biostatistics


1. Your new patient is a middle-aged man who has some deficiency in communication and social
skills but has a basic level of communication and was able to make it through grade school
without problem. IQ is found to be 65. What level of mental retardation is this?
Mild

2. A man who lost his entire family to an earthquake, reacts by donating a lot of money to
charities that help families devastated by natural disaster. What kind of defense mechanism
is this?
Sublimation

3. A student begins to feel a tightness in his chest and palpitations before giving a presentation to
a group of executives that will determine whether or not he gets promoted. He had this
feeling during 1st and 2nd midterms as well. What kind of anxiety disorder is this?
I said social phobia (performance specific).
4. A young man becomes anxious and fearful when told he is to make a presentation on stage in
front of a large audience. What kind of phobia is this?
I said social phobia (performance specific) again, though I was a bit suspicious of the same answer
coming up twice. Don’’t know what they are trying to get at with these two questions.

5. A young woman is preoccupied with her nose which she states is misshapen and goes to
various doctors to ask for it to be fixed. The physicians assure her that her nose is just fine,
but she continues to worry over it. What kind of disorder is this?
Body dysmorphic disorder.

6. What kind of hallucinations are most frequently associated with psychomotor epilepsy?B
Olfactory

A female patient has been feeling tired, has decreased appetite, difficulty in concentration, trouble
sleeping, and loss of interest in sex and life in general. This occurred for the past three weeks.
7. What is her diagnosis?
Major depressive episode.
8. How would you describe her self-esteem(sense of self-worth)?
Don’’t know exactly what they were looking for here, but I said low self-esteem (guilt).

9. A woman is brought in by her husband after she attempted suicide on herself. She states that
she feels worthless, has done this before, and is afraid that her husband is going to leave her.
What cluster of personality disorder is this?
Cluster B

10. What is the equation for specificity, using the variables for true negative (u), false positive (v),
true positive (w), false negative (x), total negative (y), total positive (z)?
u/(u+v)

MEQ 10- Endocrine


A 17 year old girl with known diabetes is brought into the emergency room due to vomiting and
decreased sensorium. Her breath smells fruity, and urinalysis comes positive for glucosuria.
1. Which ketone body is at the highest concentration level in her body?
ȕ-hydroxybutyrate
2. Which organ produces the majority of ketone bodies?
Liver
3. Which enzyme is at increased levels in the patient at this time and leads to increased
lipolysis?
Hormone-sensitive lipase

A woman presents with truncal obesity and other symptoms of Cushing’’s Syndrome. A CT scan of the
abdomen shows a nodular enlargement on the adrenal gland.
4. Which hormone is at increased levels in this patient?
Cortisol
5. Which histological zone of the adrenal cortex is aldosterone produced?
Zona glomerulosa
6. The adrenal medulla is innervated by what kind of autonomic nerves?
Sympathetic
A patient undergoing subtotal thyroidectomy accidentally has his parathyroid glands removed. After
the procedure he presents with carpopedal spasms and muscle twitching.
7. Which substance in the serum is at an altered homeostasis to present with such
symptoms?
Calcium
8. What enzyme in the kidney is responsible for activation of Vitamin D to its active form
(1,25-dihydroxycholecalciferol)?
1-Į-hydroxylase

A patient presents with episodic hypertension associated with headaches, palpitations, and diaphoresis.
He is found to have a pheochromocytoma. He also has medullary carcinoma of thyroid. Serum levels
reveal elevated levels of PTH.
9. Which type of multiple endocrine neoplasia (MEN) is this?
MEN 2A
10. What hormones secreted by the tumor are responsible for the symptoms found in this
patient?
Norepinephrine, epinephrine

MEQ 11- Neurology


A young boy is brought in with enlarged cranium. MRI shows that he suffers from Arnold-Chiari
malformation.
1. Which type of hydrocephalus does he have?
Non-communicating
2. Where does CSF travel through to get from the lateral ventricles to the 3rd ventricle?
Interventricular foramen of Monro

A man who diagnosed with medial medullary syndrome has a CT done which shows lesions in the
medial lemniscus, left hypoglossal nucleus, and left medullary pyramid.
3. What side would his tongue deviate towards if you ask him to stick it out?
Left
4. Name two sensations that would be deficient on the contralateral side of his body.
Vibratory, proprioception, touch

A man presents with homonymous hemianopia with macular sparing. Pupillary light reflex is present.
5. Where is the likely lesion?
Visual cortex
6. Where in the brainstem do the parasympathetic nuclei to constrict the pupils reside?
Edinger-Westphal nucleus in midbrain

A man with a cerebellar lesion presents with ataxia and wide-based gait.
7. What kind of tremor does he have?
Intention tremor
8. Where do the afferents to the flocculonodular lobe originate from?
Vestibular apparatus

Image shown of cross-section of spinal cord with different parts and pathways labeled with letters.
9. Identify the area of the posterior columns where the arrow is located (arrow not shown
in this image).
Cuneate fasciculus of the dorsal columns.
10. Where would a lesion have to occur to result in symptoms of upper motor neuron lesion?
Identify lateral corticospinal tracts.

Exam 3
An elderly man and known alcoholic presents to the emergency room with aberrant behavior and
decreased sensorium. What kind of hallucination would he be most likely to have?
Tactile because he is having alcohol withdrawal.

A man comes in with loss of interest and suicidal intentions. In order to be considered for a
major depressive disorder, he must have 2 out of 7 symptoms for how long?
I don't know. I thought Major Depressive Disorder was 5 out of 9 symptoms, and for 2 weeks so that's
what I chose.

Delirium is a state of decreased consciousness with decreased level of arousal as a result of


decreased levels of what neurotransmitter?
Acetylcholine, not glutamate.

Which of the following groupings of personality disorders are all in the same cluster?
This question was obviously faulty because two of the answers were correct. One grouping was
paranoid, schizoid, and schizotypal. Another one was avoidant, dependent, and obsessive-compulsive.

Some question on dementia vs. delirium, but I did not know the answer because it was on some
obscure aspect, not what was taught in class.

An old man is brought into the hospital because of several occasions of exposing his genitalia to
innocent bystanders. What is your next step in treatment?
This person is demonstrating exhibitionism but I said that I would check first for dementia. Another
answer choice was to treat him with medication, but no medications are FDA-approved for
exhibitionism, and it is treated mostly with behavioral therapy. Naltrexone and SSRI's have been used
to treat the disorder.

A diabetic woman presents because she has begun to have a tingling, burning pain in her feet and
legs. What is the pathogenesis of diabetic neuropathy?
I challenged this question because sorbitol and AGE were both answer choices, and we were taught that
all four pathways (sorbitol, AGE, PKC, and hexosamine) lead to the microvascular lesions seen in
diabetes.

Which of the following is not considered an Axis I Sexual and Gender Disorder?
I said having abnormally high levels of sexual urge. Don't think the answer is drug-induced sexual
dysfunction because this is under the sexual disorders as substance-induced.

A man believes that his landlord is using a radioactive machine next door to try to kill him and
his family. He searches online for coupons to save money. What kind of disorder does this man?
I think delusional disorder, but how do you decide that this is, and not schizophrenia. In delusional
disorder, functioning is otherwise not impaired. I guess that's what they're trying to say in that the man
continues with his daily life.

A patient with avoidant personality disorder will most likely have what?
A good job? Or a college degree? I have no idea.

A man presents to the psychiatric clinic for the first time because of recent hallucinations and
delusions that others are trying to kill him. He is diagnosed with schizophrenia. What is he least
likely to have?
Younger onset. You have to recognize this as paranoid schizophrenia, which is characterized by an
older age of onset, better premorbidity and outcome, more likely to get married and have children, and
more likely to have a job. Whereas disorganized schizophrenia has an earlier age of onset.

Which of the following is not involved in Wernicke-Korsakoff syndrome?


Hypothalamus. Not nystagmus, mamillary bodies, thiamine, or ataxic gait, as these are all involved.

Which of the following would not be involved in the diagnosis of ADHD?


I said grandiosity, but wasn't sure because I don't think irritability is necessarily a part of ADHD either.

An epileptic partial seizure usually originates from which part of the brain?
Temporal lobe, not parietal lobe.

A woman is diagnosed with selective ACTH and cortisol deficiency. What would you expect to
find on physical examination?
I don't even know how you get selective ACTH and cortisol deficiency is. Do you see ankle edema,
secondary amennhorea, or loss of pubic and axillary hair?

A woman presents with a unilateral periorbital headache. Her temples are tender on palpation,
and ESR is found to be increased. She is suspected of having temporal arteritis. If she indeed
does, what would you expect to see on biopsy of the temporal artery?
Granulomatous inflammation of intima and inner media with fragmentation of internal elastic lamina.

An obese girl with mental retardation, abdominal cramps, and short stature is found to have the
following lab findings: ĻCa, Ĺphosphate, ĹPTH. What does she have?
Pseudohypoparathyroidism. Not Wilson Syndrome, which is a thyroid syndrome involving decreased
conversion of T4 to T3, presenting with body temperature below normal, but a normal thyroid function
test.

A woman presents with dementia and spastic movements of her arms. Her father had depression
and committed suicide. She is diagnosed with Huntington disease. What is the pathogenesis of
her movement disorder?
Loss of striatal neuronsĺĻinhibition of globus pallidus externus, which is thus more active to inhibit
the subthalamic nucleusĺless activation of globus pallidus internusĺdecreased inhibition of
thalamusĺincreased motor activity

1st question: A man presents with weakness and atrophy of the right arm, fasciculations, and
hyporeflexia of the brachioradialis. He notes no sensation loss or incontinence. Where is the
lesion?
Anterior horn cell, not peripheral neuropathy because that would present with sensory loss.

2nd question: If the aforementioned man also has hyperreflexia of the triceps, where else must
there be a lesion?
Prefrontal cortex because if the brainstem were involved, he would have incontinence.

An immigrant presents with headache and nuchal rigidity. He is found to be suffering from
tubercular meningitis. What would you expect the glucose levels to be like?
Decreased. Fungal and TB meningitis present with the same findings as bacterial meningitis except for
the cell type. Viral meningitis has normal glucose, normal or increased protein, and increased
lymphocytes.

A man presents with signs of both LMN and UMN lesions. What is usually the first sign seen in
amyotrophic lateral sclerosis?
Atrophy of the intrinsic muscles of the hands, not weakness of the extensors of the legs.

What is the most frequent reversible cause of dementia?


Vascular dementia

What can differentiate a hemorrhagic versus an ischemic stroke?


I said seizure because I thought this would occur in hemorrhagic (due to blood coming in direct contact
with the neurons), and not in an ischemic stroke, but I think both types of stroke can result in seizure.
Hemorrhagic stroke occurs if there is reperfusion injury after embolus, whereas ischemic stroke is
usually due to atherosclerotic thrombus.

A man presents with bilateral target lesions on his palms of 1 day duration. They appeared after
he had been treated for a UTI using trimoxazole. What does he have?
I wasn't too sure about this because of the ambiguous wording: are they saying that he had the lesions
for only 1 day? Or that it's been a day now and he still has it? I think the answer is erythema
multiforme though because they appeared after his treatment with trimethoxazole, which includes
sulfonamides, an etiological factor for EM.

An elderly man presents with blisters on the flexor surfaces of his elbows. There is no mucosal
involvement. What does he have?
Bullous pemphigoid.

A young male presents with non-tender comedones lesions on his face and back. The
pathogenesis of his condition involves the release of what?
I challenged this question because of the wording. Triglycerides are actually what are released by the
sebaceous glands. But these are cleaved by lipases from Proprionibacterium acnes to free fatty acids,
which are the actual factors that are comedogenic and chemotactic for neutrophils.

Intro to Clinical Medicine


Midterm
A 27 year old male was brought by his family to the psychiatric clinic because of his aberrant
behavior. He is socially withdrawn and claims to have special powers. Recently he has been
wearing his grandfather's clothes around. On appearance he seems disheveled. What is your
diagnosis?
Thought this question was too ambiguous. Is it schizotypal or schizoid? Social detachment points
towards schizoid, but the magical thinking and strange appearance point towards schizotypal.

A male teenager was brought in to the psychiatric clinic because he had been acting strangely for
the past six months. He hears auditory hallucinations and claims that a chip is implanted in his
brain that tells him what to do. He is suspicious of everybody and tells you during the
appointment that his family is the enemy. He is always looking over his back as if expecting for
something to happen. What kind of schizophrenia is this?
I believe this to be paranoid schizophrenia.
There are five subtypes of schizophrenia:
1) Paranoid- preoccupation with one or more delusions or frequent auditory hallucinations, and
none of the following is prominent: disorganized speech, disorganized or catatonic behavoir, or
flat or inappropriate affect.
2) Disorganized- has disorganized speech, disorganized behavior, and flat or inappropriate affect.
3) Catatonic- motoric immobility as evidenced by catalepsy (including waxy flexibility) or stupor,
excessive motor activity (that is apparently purposeless and not influenced by external stimuli),
extreme negativism (an apparently motiveless resistance to all instructions or maintenance of a
rigid posture against attempts to be moved) or mutism, peculiarities of voluntary movement as
evidenced by posturing (voluntary assumption of inappropriate or bizarre postures), stereotyped
movements, prominent mannerisms, or prominent grimacing, echolalia or echopraxia.
4) Undifferentiated- none of previous criteria met
5) Residual- absence of prominent delusions, hallucinations, disorganized speech, and grossly
disorganized or catatonic behavior, and continuing evidence of the disturbance as indicated by
the presence of negative symptoms or two or more symptoms from Criterion A for
Schizophrnia, present in an attenuated form (e.g. odd beliefs, unusual perceptual experiences)

A 37 year old lady is brought in to the clinic after an incident of public indecency. She claims to
be appointed by God to inform the world that it is about to end. When her family was contacted,
they said they hadn't heard from her in 3 weeks and found it to be strange. The patient is
wearing a bright red dress with bright red shoes. Her speech is very rapid and hard to follow.
What is your diagnosis?
Thought this question was too ambiguous. Is it paranoid schizophrenia or delusional disorder? The
delusion and preoccupation points towards paranoid schizophrenia, but she also has disorganized
speech. Delusional disorder is 1 month of non-bizarre delusions without active-phase symptoms of
schizophrenia, which she has.

A 17 year old was brought in by his mother because he had been acting strangely and laughing
inappropriately. A drug test detects levels of marijuana in his blood, and he admits to have taken
it. What symptom would you expect to see in the patient?
I said nystagmus, but I think the correct answer is conjunctival injection. Other answer choices were
bradycardia, hypersalivation, or miosis, but instead of those you should find tachycardia and
xerostomia. Miosis is characteristic of opioid intoxication, whereas nystagmus is found in PCP
intoxication as well as sedative or inhalant intoxication.

A 45 year old African male who had just immigrated from Kenya has a recent history of suffering
from weight loss, mild fever, and night sweats. He maintains an upright sitting position to
decrease his discomfort, and upon examination, lung auscultation is clear. Fluid removed from
the pericardial sac is bloody. What is your diagnosis?
Thought this question was too ambiguous. Is it tuberculous pericarditis or purulent pericarditis? I
think tuberculous because of the hemorrhage, but then I would not expect the lung auscultation to be
clear.

A woman comes into the clinic complaining of abnormal menstrual bleeding. An ultrasound is
taken and a snowstorm pattern is seen in her enlarged uterus. What is the definitive way to
diagnose this as a hydatidiform mole?
Histopathological biopsy, ultrasound, CT scan, dilated placental villi on imaging? I cannot find the
answer to this anywhere, but I believe the correct answer to be histopathological biopsy.

A woman who has come in for a checkup is found to have malignant-looking microcalcifications
in her left breast. She is anxious because she has two friends who have breast cancer and in
advanced stage. She has no family history of breast cancer. What is your next step in treatment?
I have no idea. I said MRI of right breast just to be sure and screen for bilateral involvement but
thought the question was not fair because I also would want to do a biopsy of the lesion in the left
breast. Other choices were bilateral ultrasound or mastectomy.

A woman found a lump in her breast right before beginning an athletic competition. The lump
has ill-defined borders, is relatively immobile, and no lymphadenopathy is palpable. What is
your diagnosis?
Invasive carcinoma or carcinoma in situ? I did not like this question because the ill-defined borders
points to invasive carcinoma, but the absent lymphadenopathy points to carcinoma in situ.

A 57 year old alcoholic in the clinic claims to see little green men walking around trying to kill
him. He hasn't had a drink in two days. Needle track marks are found on his arm. What is he
suffering from?
Delirium tremens. This happens 2-5 days after last drink, and symptoms in order of appearance are:
autonomic system hyperactivity (tachycardia, tremors, anxiety, seizures), psychotic symptoms
(hallucinations, delusions), and confusion. Wernicke's encephalopathy is characterized by a triad of
confusion, ophthalmoplegia, and ataxia. Korsakoff's syndrome includes irreversible memory loss,
confabulation, and personality change (psychosis). Heroin withdrawal would present with anxiety,
insomnia, anorexia, sweating, dilated pupils, piloerection, fever, rhinorrhea, nausea, stomach cramps,
diarrhea (flu-like symptoms), yawning.

An AIDS patient with CD4 count level of <100 presents with hemiparesis and unilateral visual
loss. What other findings would this patient present with?
These findings are indicative of progressive multifocal leukoencephalopathy caused by JC virus
infection. This results in demyelination of CNS due to destruction of oligodendrocytes and would also
cause seizures, dementia, personality changes, and gait problems.

An alcoholic man presents with ataxic gait and impaired position and vibration sense. What
other findings would this patient present with?
These findings are indicative of subacute combined degneration of the spinal cord due to Vitamin B12
deficiency. In this neuropathy you may also find visual deficit, and hyperreflexia or hyporeflexia?
First Aid states hyperreflexia. Kochar states hyporeflexia. Would there be urinary incontinence due to
damage to lateral corticospinal tracts?

A 38 year old G1P0 woman with history of hypertension begins labor and is found to have a
slightly detached placenta. What gave her an increased likelihood of developing he condition she
has?
This lady is suffering from abruptio placentae, which is characterized by painful bleeding in 3rd
trimester due to abrupt detachment of placenta. Risk is increased due to hypertension (answer),
smoking, and cocaine use. Not due to age.

A G3P3 woman has just delivered her third child by spontaneous vaginal delivery, but now
returns to the hospital days later with profuse vaginal bleeding. Her first two children were
delivered via C-section. She is found to have decidualis basalis tissue in her myometrium. What
may have caused this?
This lady is suffering from placenta accreta, which is characterized by massive bleeding after delivery,
and no separation of placenta because defective decidual layer is attached to myometrium. Risk is
increased due to previous C sections (answer), inflammation, and placenta previa.

A diabetic woman who is using an IUD for family planning presents with foul-smelling vaginal
discharge. Pelvic examination reveals uterine tenderness. What is the most important factor
predisposing her to PID?
Recent insertion of IUD. Diabetes not listed as predisposing factor.

A 47 year old lady has pain in the left lower quadrant of her abdomen. This pain began after
consumption of spicy foods. No bowel sounds are heard on auscultation. Palpation of the uterus
reveals adnexal tenderness. What is your diagnosis?
I said diverticulitis, but I can't find anything on spicy foods inducing diverticulitis. Only that spicy
foods can induce labor, but how would this cause adnexal tenderness?

A woman who had surgery for cholecystitis 2 days ago now presents with dyspnea. There is
dullness on percussion at her right lower lobe, and decreased breath sounds also. What is your
diagnosis?
Thought this question was too ambiguous. Is it atelectasis or pulmonary embolism? The history of
surgery and bed-ridden state definitely points to pulmonary embolism, but I don't know how that would
present, and it can lead to atelectasis (OR pleural effusion) both of which could present with such
findings I feel.

A man suffering what he says is a ““heavy feeling”” in the heart, has chest pain on exertion which is
relieved by rest. EKG is done and shows a sinus arrhythmia. What would you expect on the
EKG findings?
The patient has normal sinus arrhythmia, so there probably is no conduction problem. I was thinking
stable angina and looking for characteristic ST depression, but alas of course they do not have this as
an answer choice. Answer choices include ventricular hypertrophy or prolonged PR interval.
According to the Pathophysiology notes (Cardiology; Coronary Artery Disease; pg. 46), during an
angina attack you could see the ST depression as well as the T wave being discordant to the QRS
complex, and a decreased R wave height. In between attacks you can see evidence of previous infarcts
(abnormal Q waves) or hypertrophy, as also stated in Kochar (pg. 207), so I feel this is the correct
answer.

A military officer who suffered from prolonged erection years ago now presents with loss of
proprioception in his legs and pupils that react to accommodation but not to light. What else
would you expect to find in this patient?
The patient has tabes dorsalis, which occurs in 3° syphilis. At this stage of disease, you may also find
gummas (chronic granulomas), aortitis, Charcot joints (degeneration), shooting pain, and absence of
deep-tendon reflexes. No maculopapular rash as that is in 2° syphilis.

A 40 year old lady goes to see the doctor because she is worried about chest pain she felt 2 months
ago. When she went to see the doctor right after that incident, tests were done and everything
was normal, and she was told not to worry. She also worries about her parents' health. This
affects her, and she is worried that she may lose her job. She has had this problem in the past
when she was younger, as she would go to the doctor a lot. She is sweating and anxious. What is
your diagnosis?
Thought this questions was too ambiguous. Is it hypochondriasis or general anxiety disorder? General
anxiety is persistent and generalized anxiety for at least 6 months, which you can say this lady has. But
she also worries about having a disease for at least 6 months despite medical attention ruling out
disease, pointing towards hypochondriasis. I feel the correct answer is general anxiety disorder
because her anxiety is not confined just to having a disease but about other things as well.

A baby in respiratory distress was treated and found to have been infected by respiratory
syncytial virus. What did the baby suffer from?
Bronchiolitis. Not bronchitis nor pneumonia.

Final
Matching question in which the patient is a teenage daughter brought in by her mother who
witnessed her have a ““seizure.”” She was shaking violently and had urinary incontinence.
Grand mal seizure, which is same as tonic-clonic: alternating stiffening and movment. Petit mal
seizure is absence seizure, which is characterized by a blank stare.

A male patient presents with hyporeflexia and fasciculations of the biceps reflex and
hyperreflexia of the triceps reflex. There is hyporeflexia of abdominal reflex and hyperreflexia in
the lower limb. There is ankle clonus and positive Babinski sign in the left foot. There is positive
Lhermitte sign. There is loss of proprioception and vibratory sensation in the C6 level but intact
spinothalamic senses. What is the diagnosis?
I wasn’’t sure and chose Guillain-Barré syndrome, but apparently Lhermitte sign is classical for
multiple sclerosis. This wasn’’t an answer choice however, and I don’’t know if Lhermitte sign is
positive in amyotrophic lateral sclerosis. The sign itself is elicited by pounding on the posterior
cervical sign while the neck is flexed. If an electrical sensation runs down the back into the limbs, the
sign is positive, suggesting a lesion in the dorsal columns.

A female patient has had vision problems and has trouble walking in a straight line. She has
diadochokinesia and a positive Lhermitte sign. She has 4+ reflex in the right leg and 2+ in the
left leg. What is the diagnosis?
I chose Guillain- Barré again because I thought maybe Lhermitte sign is indicative of this, but the
multiple sclerosis was an answer choice here, so that was the answer.

A lady who has had history of abdominal pain goes to the doctor to get it checked. After
extensive investigation, the clinician lets her know she has irritable bowel syndrome. She is not
convinced however, and believes she has a serious illness, so she continues calling and makes
several appointments with other clinics despite the results. What is the diagnosis?
I wasn’’t sure between hypochondriasis or maybe pain disorder, but hypochondriasis seems like the
more correct answer they are looking for.

A lady with discomfort in the lower abdominal area and anus also has menorrhagia. Upon
examination there is blue discoloration of the skin.
Endometriosis or adenomyosis?

A 48 year old woman is brought in by her daughter. She says that her mother is diabetic and
usually kept in control with insulin and diet, but she has recently complained of loss of appetite.
On presentation she is sweating and has cold and clammy skin. What would you find her blood
levels to be like?
I said hypoglycemia because she had lost her appetite and probably hadn’’t eaten. And/or may have
overdosed on insulin. But the signs of hypoglycemia include hunger, as well as tremor, tachycardia,
sweating, dizziness, anxiety, blurry vision, fatigue, headache, irritability. Hyperglycemia would present
with polydipsia, polyphagia, polyuria, dry skin, blurry vision, drowsiness, and slow wound-healing.

A 83 year old man presents with decreased vibratory sensation in his knees, tremor in his fingers
when extended fully, decreased postural stability when eyes are closed, and other symptoms.
Which one of these is found with normal aging?
I wasn’’t sure. Chose tremor in fingers when extended fully.

A 11 year old boy suffers from recurrent abdominal pain and nausea. Antacids do not help to
relieve the pain. This happens in the morning before school and thus has caused a lot of missed
schooldays. There is no association with vomiting, diarrhea, or bleeding. Does not disturb his
sleep. What is the diagnosis?
I wasn’’t sure. Chose abdominal migraine. Other answer choices were malrotation of the gut or
Meckel’’s diverticulum. Why must the Clinical Medicine Department put so many random questions
that they don’’t teach us about?

A man has had chronic lower back pain. He falls and injures himself. He now presents with
positive bilateral response to straight leg raise test and loss of sensation in his groin area. What is
the diagnosis?
I wasn’’t sure. Chose cauda equina syndrome, which happens to be correct. Another choice was
prolapsed intervertebral disc. Cauda equine syndrome is caused by injury to multiple lumbosacral
nerve roots, and can be caused by impingement from tumor, stenosis, inflammatory conditions, or
trauma from fracture, severe prolapsed disc, iatrogenic, etc. Symptoms are low back pain, weakness,
areflexia in the lower extremities, saddle anesthesia, and loss of bladder function.

A lady has swelling in the neck and signs of hyperthyroidism. She had an infection a few weeks
ago and removal of a breast tumor in the past. There is diffuse enlargement of the thyroid. TSH
levels are low, T4 levels are low. Radionuclide uptake is decreased. What is the diagnosis?
The low TSH as well as T4 levels confused me. I chose subacute thyroiditis, which is the most likely
answer because of the history of infection. Except one would expect to find high T4 levels because of
release of preformed thyroid hormone.

A lady is found to have a mid-diastolic murmur and loud S1 sound. Which position would be
best to listen to the murmur?
She has mitral stenosis, so the best would be left lateral decubitus position.

A 36 year old woman who is hemoglobin SS presents because of severe pain in the abdomen and
extremities. She is found to have central pallor. What other findings would you expect for this
patient?
I challenged this question because I think the sickling crisis could cause peripheral cyanosis, but would
you see hyperactive spleen and/or jaundice as well because of the extravascular hemolysis?

A lady presents with pain in the right hypochondrium after eating dinner. The pain radiates to
her scapula. She is a known alcoholic and has a history of gall stones. There is no jaundice.
What is the diagnosis?
I challenged this questions because I felt she could have either have a gall stone stuck in the cystic duct
or cholecystitis. The former would lead to the latter anyhow and present with the same symptoms.

A male patient complains of blood in his urine. His two brothers have a history of renal disease
and were told by their physician that they are to be placed on renal dialysis. The patient is found
to be obese, and both kidneys are palpable and nontender. Digital rectal exam reveals an
enlarged prostate. What is the reason for his clinical presentation?
I challenged this question because it looks like he has polycystic kidney disease, but the fact that an
enlarged prostate makes me think of BPH which could cause urinary retention leading to complications
of hydroureter and hydronephrosis.

OSCE
Physical Exam #1- Tennis elbow- forgot to do muscle tone
Physical Exam #2- Appendicitis- forgot to inspect hair distribution
History Taking #1- Erectile dysfunction
History Taking #2- Rheumatoid arthritis and Sjögren syndrome
EKG- don’’t know diagnosis, there were peaked T waves in leads V1-V3 and then inverted T waves in
V4-V6. Apparently this was supposed to be an anterior wall MI……
X-ray- Abdominal view Kidney-Ureter-Bladder, with multiple calcifications within kidney; patient
history unremarkable except history of hypertension. So I definitely thought this was polycystic kidney
disease because there were calcifications all over both kidneys, as well as the history. But I guess you
don’’t see cysts on X-ray, as this was supposed to be multiple calcifications due to multiple renal
infarcts.
MCQ-
A 37 year old lady presents to the emergency response in a state of distress and difficulty
breathing. She has immense pain in the left side of her chest that is constant and unremitting.
This occurred 15 minutes ago when she collided with someone during a game of basketball. She
denies use of alcohol, but has smoked for the past 10 years. She does not take any drugs, but is
on oral contraceptives. What is the first thing you do?
Check that her airway is open. This was tricky because I wanted to choose do an EKG, because have to
rule out an MI, but I remember from BLS that first thing to do is check airway, then breathing, then
circulation (ABC).

After making sure that her airway is open, you do arterial blood gas analysis and find that she
has 70 mmHg PCO2 (normal 100). You quickly apply a gas mask to the patient to provide
oxygen. Your initial physical exam makes you suspect that she may have a tension
pneumothorax. What do you do next?
I said this time to do an EKG because want to rule out MI, but the answer was continue with your
physical exam. Blah, arguable. Another answer choice for first and second question was Chest X-ray.

You finish your physical exam and find that indeed the patient has a tension pneumothorax.
How would tension pneumothorax present on auscultation and percussion of the left hemithorax?
Decreased breath sounds, hyperresonant.

You try to decompress the tension pneumothorax by inserting a needle into the thoracic cavity to
release the accumulated air. Where in the chest would you stick the needle?
Is it the 2nd intercostal space midclavicular line, 3rd intercostal space midclavicular line, 4th intercostal
space midaxillary line, 5th intercostal space midaxillary line? I thought the lowest the better and that
thoracocentesis should be done at the 8-10th intercostal space if midaxillary line, or 6-8th intercostal
space at midclavicular line, but I forgot that this is to remove fluid, which would accumulate at the
lowest point. Removal of air should be from the most superior location or 2nd intercostal space
midclavicular line.

Unfortunately the patient does not survive. You perform a post-mortem chest X-ray to confirm
the diagnosis of tension pneumothorax. Which of these radiographs would be representative?

Pharmacology
Exam 1
A lady with congestive heart failure is given a drug to treat her hypertension. She feels better
and is discharged to go home. A few days later, however, she comes in again complaining of
hearing loss. Which drug was given to her?
A loop diuretic, specifically ethacrynic acid.

An asthmatic patient comes in with hypertension and is prescribed a beta blocker to lower his
blood pressure. Which beta blocker would be best for this patient's use?
Selective ȕ1 blockers are recommended for asthmatics, but I got all the ȕ-acting drugs mixed up, so got
this wrong by choosing isoproterenol.
Non-selective ȕ-agonist- isoproterenol
ȕ1-agonist- dobutamine
ȕ2-agonist- terbutaline, albuterol, salmeterol, formoterol
Non-selective ȕ-antagonist- propanolol, nadolol, timolol
ȕ1-antagonist- atenolol, metoprolol, esmolol
Į1 & ȕ-antagonist- labetalol, carvedilol
partial agonist- pindolol
The ȕ1-antagonists used to treat hypertension are atenolol and metoprolol.

A diabetic patient comes in with hypertension and is prescribed medication to decrease his blood-
pressure. Which drug would be preferred for this patient?
Don't remember which of these was the answer, but could have been ACEI, ARBs, thiazide diuretics,
calcium channel blockers, or ȕ1-antagonists.

A patient is being treated for hypertension. On examination, he is found to have renal stenosis.
Which drug is contraindicated in patients with renal stenosis?
ACEI

A patient comes in with a red, inflamed, tender 1st metatarsalphalangeal joint. He is diagnosed
with an attack of acute gout. What would be your first-line treatment in this case?
I wasn't sure between naproxen or prednisone because they are an NSAID and glucocorticoid,
respectively, both of which are used in cases of acute gout, but NSAIDs are first-line.

Adrenergic drug study in which Drug X causes increase ĹHR, with hexamethonium causes ĻHR,
and with scopolamine causes has no effect.
I chose nitroglycerin because I was confused and forgot scopolamine was anti-muscarinic. Thought it
was a serotonin-acting drug. The answer was most likely acetylcholine or another cholinergic agonist.

Question came with accompanying diagram of a nephron and you had to identify which part of
the nephron acetazolamide act on.
I thought this was a sketchy question because there was a choice of proximal convoluted tubule or
proximal straight tubule. The notes just mention proximal tubule, but the diagram in the notes sort of
suggest proximal convoluted tubule as the answer.

A patient with congestive heart failure is prescribed medication to treat her hypertension. Which
calcium channel blocker should be avoided as it could worsen the heart failure?
Verapamil, as it has a negative inotropic effect and is the least selective of the calcium channel blockers
(significant effects on both cardiac and vascular smooth muscle tissue). Diltiazem affects both cardiac
and vascular tissue but more so vascular, whereas nifedipine and amlodipine affect primarily the
vascular Ca2+ channels.

A patient suffering from atrial fibrillation as well as heart failure is on hydrochlorothiazone,


enalapril, and digoxin.. Metoprolol is added to her drug regimen. What would you expect in one
week?
The drug of choice for rate control of atrial fibrillation in a patient with heart failure is digoxin.
Metoprolol is used to treat heart failure and decrease the amount of cardiac remodeling that occurs, but
it has a negative inotropic effect on the heart and thus you would see an initial worsening of the heart
failure. Because of this, it is contraindicated in patients with decompensated heart failure.

Which of these drugs would lower blood pressure while concurrently decreasing plasma renin
levels?
A beta blocker because of its actions on ȕ1. So either propanolol, metoprolol, atenolol, or pindolol
would have been the correct answer.

Which of these drugs is the drug of choice for for trigeminal neuralgia?
Wasn't sure between TCA and anticonvulsants because both are used for neuropathic pain, but
anticonvulsants, namely carbamazepine is the drug of choice for trigeminal neuralgia.

Tramadol is a mild to moderate opioid agonist used to treat neuropathic pain. It acts as a weak ȝ
agonist on opioid receptors, and also acts to:
Block reuptake of serotonin and norepinephrine.

A 78 year old lady presents with urinary incontinence. A drug is prescribed to treat her
symptoms. Activation of which receptor would cause contraction of the detrusor muscle?
I was confused by this question because the detrusor muscle is what contracts to push and help in
voiding urine, so I don't think a medication that does this would be helpful in treating urinary
incontinence. However, a drug that causes contraction of the sphincter would help to prevent
incontinence. But to answer the question, a drug that causes contraction of the detrusor muscle would
activate muscarinic receptors.

Which of the following parameters is constant in first-order kinetics?


Clearance. Thus since rate of elimination = CL x C, elimination is directly proportional to drug
concentration, and a constant fraction of drug is eliminated per unit time. Half-life is also constant in
first-order kinetics. In saturation ““zero-order”” kinetics, a constant amount of drug is eliminated per unit
time, and this clearance varies with drug concentration.

A patient is taking an opioid drug to treat his chronic pain condition. Which of the following
opioid-induced side effects will take the longest to develop tolerance to?
Constipation. Tolerance develops slowly if at all. Nausea/vomiting tolerance develops within a few
days. Same with sedation. Tolerance to respiratory depression develops quickly.

Exam 2
A pregnant woman going into premature labor is given a drug to delay the delivery. After taking
the drug, she starts to develop palpitations, anxiety, and tremulousness. Which drug did she
take?
I wasn’’t sure between magnesium sulfate, nifedipine, and terbutaline because they seem to have
overlapping side effects. I chose magnesium sulfate, but the answer is terbutaline.
MgSO4- flushing, nausea, headache, drowsiness, blurred vision
Nifedipine- tachycardia, palpitations, flushing, headaches, dizziness, nausea
Terbutaline- palpitations, tremor, nausea, vomiting, nervousness, anxiety, chest pain, shortness of
breath, hyperglycemia, hypokalemia, hypotension

A 35 year old woman who is taking ibuprofen for her osteoarthritis is also given a drug to prevent
NSAID-induced ulcers. After taking this drug however, she begins to have abdominal cramping.
Which drug did she take?
I chose misoprostol because it has a side effect of inducing uterine contractions via its prostaglandin
activity, but I challenged the question because another answer choice was magnesium hydroxide, which
I argued could cause diarrhea which the patient could describe as abdominal cramping, or sucralfate
which contains aluminum hydroxide, can cause constipation, which could also be described as
abdominal cramping. Misoprostol was the correct answer.

A 33 year old woman taking medication to treat tuberculosis is also given pyridoxine. Why is she
given pyridoxine?
To prevent neural toxicity caused by isoniazid.
A 37 year old male on anti-tuberculosis medication begins to complain about red-green color
blindness. Which drug can cause such adverse effects?
Ethambutol

A 54 year old male on medication to treat tuberculosis calls his physician because his urine has
turned bright orange, and he is afraid he has hemoglobinuria. This side effect can be caused by
which drug?
Rifampin

A patient is prescribed amoxicillin to treat an infection. What is true about amoxicillin?


I wasn’’t sure between has a better oral bioavailability compared to ampicillin, or does not have
hypersensitivity reaction. I chose that it has better oral bioavailability because the notes state that both
are acid-stable and well-absorbed, but also that oral absorption is impaired by food, with exception to
amoxicillin. But the notes also state that both have maculopapular rash as an adverse reaction, but Dr.
Clunes mentions in class that this not necessarily a hypersensitivity reaction, but then again,
amoxicillin is a penicillin, to which many people have hypersensitivity reactions to.

Two questions were about patients with endocarditis. One was given to treat infection, the other
was given for prophylaxis during oral surgery. Which drugs were prescribed?
If the infective organism is streptococci, which it is most of the time, then it can be treated with
penicillin G plus gentamicin or ceftriaxone plus gentamicin. If it is MRSA or coagulase-negative
staphylococci which frequently infect prosthetic valves, or penicillin-resistant streptococci, or if the
patient is allergic to penicillin, then vancomycin plus gentamycin is given. Rifampin also added in
treatment of endocarditis if prosthetic valve. Linezolid or daptomycin can also be used for
vancomycin-resistant organisms. If the infective organism is staphylococci, then nafcillin or oxacillin
with gentamicin, or if allergic, cefazolin with gentamicin. Prophylaxis for infective endocarditis during
oral surgery is amoxicillin. If allergic to penicillin, then clindamycin, cephalexin, azithromycin,
clarithromycin. If unable to take oral medications, then ampicillin IV or IM. If cannot take orally and
allergic, then clindamycin IV or cefazolin IV.

A child presents with a stiff neck and headache, and is found to have meningitis due to Neisseria
meningitidis. What drug would be given to his family and close contacts as prophylaxis?
I chose Penicillin G because it is used to treat Neisseria meningitides, but rifampin is actually used as
prophylaxis for close contacts of a patient with meningitis caused by meningococci.

A 47 year old man gets an abdominal peritoneal infection, and the organism is found to be a
Gram-negative, ȕ-lactamase producing bacteria. The patient has a history of penicillin allergy.
What is your drug of choice for treatment?
Aztreonam because it is a monobactam which targets aerobic Gram-negative rods, is resistant to ȕ-
lactamases, and has little cross-reactivity with other ȕ-lactam antibiotics.

A child comes in with flu-like symptoms, and the physician prescribes an empirical therapy of
several drugs to cover viral, bacterial, and fungal infection. After induction of therapy, the child
presents with acute renal failure. Which drug can cause this reaction?
Amphotericin B

An HIV-positive patient with hemorrhagic enterocolitis is on ganciclovir treatment. If he is also


started on zidovudine, what side effect would you expect?
Neutropenia. Wasn’’t sure about the other answer choices of hepatotoxicity and nephrotoxicity, but
zidovudine doesn’’t cause these reactions.

A patient is diagnosed with non-small cell lung cancer and is given erlotinib to treat the cancer.
Erlotinib’’s mechanism of action is to:
Act as a reversible inhibitor of the EGFR tyrosine kinase. Wasn’’t sure about the other answer choices
of monoclonal human antibody of EGFR or reversible competitive antagonist at the EGFR, but it is not
these.

A male patient has a brain tumor removed, and it is found to be a glioblastoma. What drug
would the patient take to prevent cancer relapse?
I put prednisone because I thought the focus of the question is what kind of drug prevents cancer
relapse, but I think the question is actually focusing on what kind of drug treats CNS tumors, in which
the answer would be the nitrosureas carmustine. Other nitrosureas are lomustine and semustine.

A patient diagnosed with malaria begins taking an antimalarial drug, but soon afterwards
presents with macrocytic anemia. Which drug did he most likely take?
Proguanil. I chose artemisin, but it is not a folate synthesis inhibitor. Other drugs that inhibit folate
synthesis are pyrimethamine and sulfadoxine.

An 18 year old girl taking medication for a recently diagnosed illness, goes to a party, but
becomes flushed and nauseous after just one drink. What illness was she most likely treating?
To solve this question, which drug has disulfiram-like effects. The cephalosporins cefamandole,
cefoperazone, and cefotetan do, as well as metronidazole. One of the answer choices was pharyngitis,
so I don’’t know if the cephalosporins can treat this, but metronidazole definitely treats trichomonas
vaginalis, which was the answer. Another answer choice was vaginal candidiasis. Remember that one
should also not take alcohol when taking the antihelminthic niclosamide. Also, the antiretroviral
protease inhibitor lopinavir contains ethanol, so avoid disulfiram if taking this drug.

A 38 year old woman presents with fever, headache, malaise, and diarrhea. She is found to have
been infected with Influenza A, and her condition deteriorates drastically. She is being
maintained on ventilatory support. The physician prescribes a drug to treat her infection.
Which one should he prescribe?
I challenged this question because the answer choices included both zanamivir and amantadine.
Zanamivir is a neuraminidase inhibitor which treats both Influenza A and B, and I remembered that
amantadine works against Influenza A too, but I wasn’’t too sure about that, so I chose zanamivir. I
think the answer they were looking for was amantadine because zanamivir causes adverse respiratory
effects, which one probably should avoid in this patient on ventilatory support, but the correct answer
was actually zanamivir. Amantadine has widespread resistance and so is no longer recommended for
treatment for influenza, so zanamivir or oseltamivir is the drug of choice.

A teenage girl has been recently prescribed a drug to treat her acne. She takes the drug but
begins to break out in blisters on her face and parts of her body exposed to the sun. What drug
did she take?
I have no idea. I chose doxycycline because I think that is used to treat acne……

Exam 3
A diabetic patient is on insulin therapy but the HbA1C levels are still above the desired limit.
Pioglitazone is added to the drug regimen. What needs to be monitored while she is taking
pioglitazone?
I said liver function because FDA requires monitoring of liver function with Tzd therapy, but another
answer choice was ejection fraction, which I would argue should be checked too because Tzds can
cause or exacerbate CHF.

A man with diabetes, hyperlipidemia, hypertension, and heart failure begins to have symptoms of
thyroiditis. Which drug in his regimen may have caused this thyroiditis?
I challenged this question because I don’’t think we learned of any drugs that cause thyroiditis. I chose
amiodarone because that is the only one that has some sort of relation to the thyroid, but it just inhibits
the peripheral conversion of T4 to T3. Apparently amiodarone was the answer.

A lady comes in complaining of excessive growth of hair on her face. Her periods are regular,
and her last menstrual period was 2 weeks ago. What would you use to treat her?
Spironolactone, since it is a mineralocorticoid antagonist as well as an androgen antagonist. I just
wasn’’t sure because clomiphene was an answer choice, and I forgot what it did, so maybe it was a
synthesis inhibitor which can also be used to treat hirsutism (metyrapone). Clomiphene is actually a
SERM which act as an estrogen antagonist in the hypothalamus and anterior pituitary, effectively
preventing feedback inhibition, leading to increased release of GnRH to induce ovulation in those who
are infertile.

A female patient is prescribed a glucocorticoid. Weeks later she presents with a 20-pound weight
gain, and she is also found to be hypernatremic and hypokalemic. Which corticosteroid was she
most likely prescribed?
I challenged this question because I know that fludrocortisone has a big salt-retaining effect, but they
are considered mineralocorticoids, so if they are looking for glucocorticoids, then it must be prednisone
or triamcinolone. Triamcinolone has zero salt-retaining ability just like dexamethasone, so if
fludrocortisone is not eligible, then the answer is probably prednisone, which it was.

An elderly man comes in complaining of tremor in his hands and trouble initiating movement.
He has had Parkinson’’s disease for a year and his current treatment is not working to keep these
symptoms in check. After a full patient history and physical examination, the doctor prescribes
pramipexole. What is the mechanism of pramipexole?
Agonist at dopamine receptor. Other dopamine receptor agonists are bromocriptine, ropinirole,
rotigotine, and apomorphine (used in rescue therapy).

A man is brought in by his wife because he had trouble swallowing food last night. He has a past
history of twitching in his face and tongue, as well as other parts of the body. He is diagnosed
with amyotrophic lateral sclerosis. A drug with what mechanism of action should be prescribed
to him to prevent these muscle spasms?
I chose agonist at GABAB receptors (baclofen), but this does not have any indication for treatment of
ALS. The answer must have been central Į2 agonist (tizanidine) or inhibition of glutamate
transmission in CNS (idrocilamide, riluzole), but I don’’t remember either of these as answer choices.

A chronic alcoholic has been trying to quit but has had difficulty. What drug would help him to
prevent having another relapse?
Naltrexone. It is an opioid antagonist so I thought it is only used to treat opioid dependence, but
apparently alcohol dependence too, to reduce cravings.

A patient is given buspirone for treatment. What can be said about buspirone?
I said that it has significant sedative effects but challenged because even though it is a sedative, I
remember it being said in class that it does not work too well. It also lacks hypnotic effects, so that was
probably the answer. Another answer choice was that it has muscle relaxing properties, but it does not.

A patient is given thiopental for induction of anesthesia in a surgical procedure on the uterus.
Shortly after induction, what will happen to the thiopental?
It will be redistributed from the brain to other tissues, but hepatic metabolism is required for its
elimination from the body.

A patient is being prepared for surgery and the doctor chooses to use an IV anesthetic that has
significant negative inotropic effects on the heart. Which drug did he choose?
Propofol

Sevoflurane is widely used as a general anesthetic because of its rapid and smooth induction and
rapid recovery. Why does sevoflurane have such a faster duration of action than other volatile
halogenated hydrocarbons?
Gets degraded by hepatic enzymes?

A schizophrenic patient is on haloperidol treatment but complains that it gives him tremor and
other extrapyramidal symptoms. He is switched to thioridazine, which helps to improve the
symptoms. What characteristic of thioridazine allows it to have less extrapyramidal symptoms?
It’’s enhanced anticholinergic activity. Not due to its enhanced antagonism at Į1 receptors, though it
does have this characteristic, which leads to orthostatic hypotension.

A schizophrenic patient on haloperidol complains about having extrapyramidal symptoms. He is


switched to an atypical antipsychotic which causes little sedation and EPR side effects. This drug
also does not require monitoring of CBC. Which drug is this?
Olanzapine. Not clozapine because that causes agranulocytosis.

BSCE II
First Half (Terms 1-3)
An elderly man suffers a stroke and is brought to the hospital where he is being maintained on
life support. His three adult children are there as well and are unsure of how they would like
their father to be treated. The eldest son turns to you, the physician, and says, ““Doctor, we trust
you know what’’s best for our father. We’’ll agree to whatever decision you make as to what’’s best
for him.”” If you agree to this, it is an example of what kind of bioethical principle?
I chose deontological, but I think the answer is paternalism. Deontological theory, also called
Kantianism, is duty-based in which there are things that are inherently wrong, and a physician should
never do any of these as under no condition would those actions ever be right or moral, even if they
bring about positive consequences for others. Physicians have a moral to fulfill obligations and duties,
and to always remain truthful to the patient no matter what. Paternalism is a doctor-patient relationship
model, in which the physician is a father figure and makes the decisions for the patient, even overriding
the patient’’s wishes if he/she considers it in the patient’’s best interest.

A patient is in critical condition and prognosis looks poor. You have the opportunity to start the
man on renal dialysis, but it does not appear as if it will improve the outcome. What would be
the most bioethical thing to do in this case?
I said to recommend against starting renal dialysis, but that seems too harsh, and the other answer
choice to recommend beginning renal dialysis seems futile.
A teenage girl approaches you for counseling as she is worried about the possibility of contracting
HIV, and she wants to discuss with you how to prevent HIV. What is the best way to prevent
HIV?
I said to address her concerns about HIV, but I don’’t know if that necessarily prevents HIV. Another
answer choice was to tell her that condoms are the best way to prevent HIV.

Which of these codes of ethics has been adopted into practice in the US?
Declaration of Helsinki or Belmont Report? The Declaration of Helsinki, which is used for research
ethics guidelines, is produced by the World Medical Association(WMA), an organization the US is a
part of. The Belmont Report was adopted in the US after news of the Tuskegee study broke out, and
lead to the establishment of Institutional Review Boards to protect human subjects of research.

Which of these facts about the cardiovascular system is not true?


I thought all the answers were true. Anastamosis connects two different arteries so that if there is
obstruction in one of them, there will still be sufficient bloodflow from the other artery to supply the
tissue. Sphincters within the capillary system can close to shunt blood from pre-capillary arterioles to
post-capillary venules. All superficial veins drain into deep veins, which I was least sure about but still
think is true, so wasn’’t sure.

Which of the bones of the skull is most closely associated to the confluence of sinuses?
Occipital bone

The greater splanchnic nerves arises from which spinal cord levels?
T5-T9. Lesser splanchnic nerves arise from T9-T10, whereas least splanchnic nerves from T12.
Greater splanchnic travels to celiac ganglion and provides innervation to foregut (organs supplied by
celiac trunk), lesser splanchnic travels to aorticorenal ganglion and innervates midgut (supplied by
superior mesenteric artery), and least splanchnic travels to renal plexus and innervates kidneys and
upper ureter. Hindgut (supplied by inferior mesenteric artery) is innervated by lumbar splanchnic
nerves from spinal cord levels L1-L2.

A lady who is ataxic and stumbling, stubs her big toe and is in pain. Which dermatome provides
the sensation of pain that she is feeling?
L4

The liver separates the ventral mesoderm into which two segments?
Falciform ligament and lesser omentum, which consists of the hepatogastric ligament and
hepatoduodenal ligament. The greater omentum connects the greater curvature of the stomach (inferior
side) o the duodenum. The dorsal mesoderm develops into the gastrosplenic ligament and splenorenal
ligament.

The rib that comes in contact with the 3rd and 4th vertebral body and makes a joint connection
with the 4th transverse process is which rib?
4th rib, making costovertebral and costotransverse joints.

Postganglionic parasympathetic nerves which supply the parotid gland come from which
ganglion?
Otic ganglion, with preganglionic fibers coming from glossopharyngeal nerve and entering ganglion
via lesser petrosal nerve. Oculomotor nerve supplies parasympathetics directly to ciliary ganglion for
parasympathetic innervation of the eye, while facial nerve supplies parasympathetic innervation to the
pterygopalatine ganglion via the greater petrosal nerve for innervation of lacrimal gland, and mucous
glands of the nasal cavity, maxillary sinus, and palate. Facial nerve also supplies parasympathetics to
the submandibular gland via the chorda tympani to lingual nerve, and is responsible for salivation from
the submandibular and sublingual glands.

A premature baby is found to have a patent ductus arteriosus. What is true about patent ductus
arteriosus?
Two answer choices were that it normally closes during the Xth week of development, or that it closes
at birth, but I didn’’t choose either because it is supposed to close a bit after birth (not at birth) due to
increased oxygen tension. I chose that the ductus arteriosus arises from the 4th pharyngeal arch artery,
but this is not correct, as it comes from the 6th left pharyngeal arch artery. I think the answer they are
looking for though is that the PDA normally closes at birth because it does constrict then, but remains a
little bit patent and continues shunting blood for a few days.

What is the embryological basis for non-paralytic strabismus?


Defect in position of the two eyes in relation to one another.

Which of the following statements about lipid metabolism is true?


The only answer choice I remember is that all lipids within the body can be synthesized from stearic
acid(18:0, Ȧ-9), which is false because linoleic acid(18:2, Ȧ-6) and Į-linolenic acid(18:3, Ȧ-3) cannot
and are thus essential fatty acids. Stearic acid may be used as a precursor, however, for oleic acid(18:1,
Ȧ-9).

Which of the following molecules does not contain a high-energy phosphate bond?
AMP- it has a low energy phosphate bond, whereas creatine-phosphate, pyrophosphate, ATP, and ADP
all have high-energy phosphate bonds.

Which of the following properties of beta sheets is not true?


They may join together to form structures called leucine zippers. This is false because leucine zippers
are formed from two alpha helixes. Beta sheets do, however, have both parallel and anti-parallel
sheets.

A decrease in rate of the urea cycle would lead to what change?


I chose that there would be an increase in Į-ketoglutarate, but there would actually be a decrease
because the increased ammonium levels deplete the Į-ketoglutarate by combining to make glutamate,
which if another ammonium group is added would result in glutamine, which is seen at high levels in
disorders of urea cycle.

Which of the following is true about the reaction involved in heme degradation?
Heme degradation involves an oxidation reaction which breaks apart the pyrrole rings. This also
results in release of ferric iron and one molecule of carbon monoxide per each reaction as well as
consumption of an NADPH.

Which of the following diseases is due to deficiency of apoproteins, resulting in decreased


chylomicron levels?
Abetalipoproteinemia, which has deficiency of both apoB-100 and apoB-48, so inability to synthesize
lipoproteins and decreased LDL and chylomicrons. Beta lipoproteins are the same as LDL, so the
name abetalipoproteinemia makes sense. Alpha lipoproteins on the other hand are HDL. I went with
Tangier disease, however, because I remembered the name from biochemistry and that it had to do with
lipoproteins as well. But in Tangier, the deficiency is in ABCA1 which regulates cholesterol transport
out of cells. With reduced cholesterol export, Apo-A1 on HDLs are unable to pickup cholesterol and
are thus degraded, leading to very low HDL levels.

Which of the following is true about the lac operon?


lacZ gene makes ȕ-galactosidase(breaks lactose into glucose and galactose), lacY gene makes
permease(actively transports lactose across cell membrane), and lacA gene makes
transacetylase(acetylates galactose). If lactose is present, some will be converted to allolactose which
can bind to the lac operon repressor encoded by the lacI gene, preventing the repressor from binding to
the operator region lacO. The binding of the repressor to the operator region is termed negative control
whereas binding of an activator to the promoter is termed positive control. When glucose is not
present, cAMP is produced, which binds to the activator protein CRP (cAMP receptor protein), which
then binds to the promoter to activate transcription for lactose metabolism. But when glucose is
present, it is preferentially metabolized; production of cAMP is stopped and the CRP cannot bind to
activate transcription. Thus, lactose needs to be present while glucose absent in order for the lac
operon to be in the on position (repressor not bound and activator bound). If glucose is present, no
activator will be bound. If lactose is absent, the repressor will be bound.

A teenage girl is brought into the emergency room in a stuporous state after severe vomiting. Her
breath is noted to have a fruity odor, and urinalysis reveals presence of glucose. What is the best
genetic analysis to determine if she has diabetes mellitus type 1?
I was confused with this question because I know diabetes mellitus type 1 has association with HLA-
DR3 and HLA-DR4, but no answer choice included this. Additionally, diabetes mellitus type 2 has
more of the genetic predisposition. So I just chose chromosomal analysis.

How would you diagnose toxoplasmosis?


You diagnose via serology for IgG and IgM, or imaging of chorioretinitis and hydrocephalus, but I
don’’t remember these being answer choices. I chose to do stool testing for toxoplasma oocysts, but I’’m
not sure if humans pass the parasite in stools. Oocysts are shed by cats and transmitted to humans that
way.

Paroxysms in patients with malaria correlate to what pathological event occurring in the body?
Malarial paroxysms are the intermittent febrile attacks patients with malaria get due to simultaneous
rupture of infected erythrocytes and concomitant release of merozoites, antigens, and waste products.

According to the WHO, what is the biggest drug problem in the U.S.?
Marijuana or alcohol?

Rheumatoid factor is composed of which chains?


Heavy? Light? Kappa? Lambda? Mu? All I know is that it is IgM against the Fc portion of IgG.

Which of the following symptoms would be found in lateral medullary syndrome (Wallenberg’’s
syndrome)?
Wallenberg’’s syndrome occurs as a result of occlusion of the posterior inferior cerebellar artery. This
causes loss of:
Ipsilateral facial pain and temperature sensation due to lesion of spinal nucleus of the trigeminal nerve.
Contralateral body pain and temperature sensation due to lesion of anterolateral system.
Ipsilateral ataxia and uncoordination due to lesion of inferior cerebellar peduncle.
Vertigo, diplopia, nystagmus due to lesion of vestibular nuclei.
Hoareseness, dysarthria, ipsilateral dysphagia, Ļgag reflex due to lesion of efferents of cranial nerves
IX and X.
Ipsilateral Horner’’s syndrome due to disruption of hypothalamo-spinal fibers.

A lady who recently had a stroke begins to stumble into things on her left. The left side of her
body also seems unkempt, while the right side of her body is well-groomed and taken care of.
Which part of her brain did she most likely have her stroke?
This is a case of spatial neglect syndrome with lesion in the right parietal cortex, causing her to neglect
the contralateral left side of her body and visual field. Usually only left-sided neglect occurs because
of processing only by the right hemisphere, whereas the right visual field is processed bilaterally.

A drug is found to increase the potassium conductance across the membranes of a neuron. How
would this affect the action potential?
Action potentials travel along neurons first via sodium entry causing depolarization, later followed by
potassium exit yielding repolarization. I am unsure of how this affects the action potential. Makes it
faster, slower?

Which area of the brainstem does cranial nerve VI originate from?


The ponto-medullary junction, which is delineated by the inferior pontine sulcus, not the superior
pontine sulcus.

What molecular changes are necessary in neurons for the formation of memory?
Increased number of AMPA receptors, not NMDA receptors, in the postsynaptic membrane. Long-term
potentiation is most-related to memory formation and occurs when CA1 neurons in the hippocampus
are excited by a high frequency train of impulses along the Schaffer collateral axons of a CA3 neuron.
The CA1 becomes more sensitive to the excitatory transmitter glutamate, due to increased conduction,
insertion of new AMPA receptors postsynaptically, and later on increased number of synapses between
CA3 and CA1. Activation of NMDA receptors is needed for LTP as well, and this occurs via the
synaptic depolarization unblocking a magnesium ion from the NMDA receptor, allowing calcium to
enter the cell and activate kinases to induce LTP. If impulses arrive at low frequency however, not
enough calcium enters the postsynaptic neuron and instead phosphatases are activated, leading to long-
term depression (LTD).

When a patient is in hypovolemic shock, there will be widespread vasoconstriction to maintain


adequate blood pressure and perfusion to vital organs. However, vasodilation will occasionally
occur to allow perfusion to those areas of tissue. What drives these momentary vasodilations?
Buildup of metabolic byproducts, not localized increases in pressure.

Increased levels of which neurotransmitter would stimulate the adrenal medulla?


Acetylcholine, which acts on nicotinic receptors in the adrenal medulla to induce secretion of
epinephrine and norepinephrine.

The ST segment is when the ventricles are depolarized and in an isoelectric state. This
corresponds to which stage of the ventricular myocyte action potential?
Stage 2

You are performing some experiments and your findings have a p value of less than 0.05. What
does this tell you about your results?
The p-value is the probability of getting a test statistic more extreme than what you observed by chance
if H0 were true (there is no association). So lower p values mean that it probably was not by chance. In
other words, it is statistically significant, which is deemed when p<0.05. Thus you can reject the null
hypothesis, which states that there is no association. More likely than not there is some association of
the statistical findings you got.

A paired t-test shows that the results of your experiment have a test statistic value of 2. Assuming
the null hypothesis to be true, what is the probability of getting a test statistic within the range of
your results?
95%. Within 1 standard deviation is 68%, 2 is 95%, and 3 is 99%.

Which of the following has the most profound effect on how difficult it is to achieve extinction
after learned behavior?
Variable pattern of reinforcement. Behaviors are learned more slowly this way, but are also more
resistant to extinction, whereas continuous reinforcement yield behaviors that are learned quickly and
extinguished quickly.

Second Half (Terms 4-5)


Which of the following microorganisms is most associated with burns?
Pseudomonas aeruginosa

Which of the following mechanisms is used by bacteria to acquire and transfer genes for
resistance to antibiotics?
Conjugation via plasmids, transduction via viruses, transformation via loose DNA, or transposition via
transposons (jumping genes).

Which of the following is not a correct pairing of toxin with mechanism of action?
Tetanus toxin does not bind to a G-protein subunit and cause continuous activation. It blocks the
release of inhibitory neurotransmitters GABA and glycine in the anterior horn cells. The cholera toxin
is the one that ADP-ribosylates the Į-subunit of Gs protein, inhibiting the GTPase activity and leading
to constitutive activation, increasing PKA activity and thus cAMP, which continuously activates CFTR
to secrete chloride ions and thus water.

In your notes you had a graph that compared Fasting Man with Fasting Man Adapted, so you
should understand well the differences between the two. What would you expect to see in Fasting
Man Adapted?
I don’’t remember a graph comparing Fasting Man and Fasting Man Adapted and thought that if
anything, it would be in the nutrition notes, but still don’’t find anything. I did find graphs in a research
article on ““Insulin and Fuel Homeostasis”” from 1968:
I think the answer choice I chose was that there would be no ketonuria, because I thought that would
only occur in diabetes, but I suppose fasting and diabetes are quite similar because the cells are
glucose-starved in both, and so Fasting Man Adapted will have high levels of ketone bodies and
resultant ketonuria.

Which of these fatty acids is found in high concentrations in olive oil and canola oil?
Monounsaturated fatty acids

Predict what effect an increase in fiber in the diet would have?


Increased fiber decreases glycemic index, and glucose load = glycemic index * amount of
carbohydrate, so increased fiber would thus decrease glucose load.

What kind of an effect would increased fiber have on bile acids?


Dietary fiber binds bile acids and thus prevents their reabsorption. This is the mechanism of action
used by bile acid sequestrant resin drugs to treat hyperlipidemia.

Which of the following is a pathological change seen in early stages of rheumatic heart disease?
I was confused by this question because both Anistchkow cells and Aschoff bodies were listed as
answer choices, and these are both found early on. If anything though, Aschoff bodies is
pathognomonic. They consist of a central zone of degenerating, hypereosinophilic ECM infiltrated by
lymphocytes, occasional plasma cells, and plump activated macrophages called Anitschkow cells,
which have abundant cytoplasm and central nuclei with chromatin arrayed in a slender, wavy ribbon
(so-called caterpillar cells); these activated macrophages can also fuse to form giant cells.
Question described a clinical vignette of a young girl with a rash, and for you to make a
diagnosis.
Atopic dermatitis or allergic urticaria? I don’’t even know what the difference is between these two.

Which of the following diseases is not due to a dysfunction of cytoskeleton or filaments?


Chronic granulomatous disease. This is due to X-linked recessive lack of NADPH oxidase and thus
Ļreactive oxygen species, whereas Chédiak-Higashi syndrome is autosomal recessive defect in
microtubular dysfunction and Ļphagocytosis. Mallory bodies are intracytoplasmic eosinophilic
inclusions made of intermediate filaments and other proteins, often seen in alcoholic liver disease.

Which of the following pathological changes would you see in constrictive pericarditis?
I thought hepatic sinusoidal dilatation because of backflow of blood, but what about Mallory hyaline?

A child is found to have peppery lesions in his bones on X-ray, and high urinary levels of VMA.
What is your diagnosis?
I don’’t know…… I said neuroblastoma because it’’s a childhood tumor, but you get increased levels of
HVA, not VMA.

Which type of collagen is laid down first in wound healing?


Type III first, then Type I.

A patient presents with a painless lump in the palm that has been progressively enlarging, and
prevents him from making a full fist. What do you tell the patient?
This is a non-classical finding of the disease, this is a non-malignant disease, or it is a result of
abnormal tissue regeneration? I was confused by this question because I had no idea what the patient
has. I was thinking keloids, because it is due to abnormal tissue regeneration, and it would be a non-
classical finding of the disease, and it is non-malignant as well, so all the answers are good. I think it
might be Dupuytren’’s contracture though, which this would be a classical finding, is non-malignant,
and is due to thickening of the connective tissue within the hand.

Which of the following statements about the initiation, promotion, and progression of cancer is
true?
Initiation is quick and due to exposure to an initiating agent like a mutagen. Promotion is gradual and
due to more prolonged exposure to a promoting agent like a mitogen, with cell proliferation
propagating the initiated damage and rise of an altered clone of cells. Progression of cancer occurs
when a loss of growth control occurs as a result of further genetic alterations and genetic instability.

A patient is found to have endometrial cancer. Which of the following in her history would have
put her at more risk of developing endometrial carcinoma?
Of course I was looking for the classic risk factors of hormone replacement therapy, obesity, diabetes,
hypertension, nulliparity, late menopause, but alas none of these were answer choices. History of renal
failure? That was the only answer choice that seemed the least incorrect to me, so I don’’t know.

Which of the following is a non-malignant lesion?


Paget’’s disease of the breast, not sebaceous wen, spider angioma, nor café au lait spot.

Which of the following is a form of mononeuritis multiplex?


I think I chose carpal tunnel syndrome because I thought mononeuritis meant that only one nerve is
affected, but this is a misnomer. In actuality it involves damage to at least two separate nerve areas. It
can be caused by any vasculitis or connective tissue disease such as rheumatoid arthritis and lupus, or
diabetes mellitus. Less common causes include amyloidosis, blood disorders such as hypereosinophilia
and cryoglobulinemia, Lyme disease, leprosy, sarcoidosis, Sjogren syndrome, and Wegener’’s
granulomatosis.

A man comes in with pain in the joints. Which additional finding would help you to come to a
diagnosis of Reiter’’s syndrome?
I was confused with this question because I thought Reiter’’s arthritis consisted of uveitis, urethritis, and
arthritis. The answer choices were conjunctivitis and urethritis, or balanitis and uveitis. Apparently
though, conjunctivitis also occurs in Reiter’’s arthritis.

What structure is at risk of being damaged if a tracheotomy is done too low?


I said inferior thyroid artery, but some sources say right brachiocephalic trunk. Is the question asking
about damage done from the incision or from the tracheotomy tube?

The answer to this question was Kussmaul breathing due to diabetic ketoacidosis, but I forgot
what Biot’’s breathing is characterized by?
Biot’’s (ataxic) breathing has unpredictable irregularity; it may be shallow, deep, or stop for a short
period, and is due to respiratory depression and damage at the medulla.

Which of the following antihypertensive medications is also useful in treating kidney stones?
Chlorthalidone, a thiazide diuretic, because it decreases calcium excretion at the distal convoluted
tubule tubules, while increasing sodium and potassium excretion.

A man comes in because he woke up this morning with a swollen, red toe that was quite painful.
You prescribe colchicine to treat his gout. What is a side effect you may expect to find after
initiation of drug therapy?
I said thrombocytopenia but am not sure if this was the answer they were looking for. Colchicine may
cause aplastic anemia, so I would think so. Other adverse effects are nausea, vomiting, abdominal
pain, diarrhea, myopathy, neutropenia, and alopecia. Also should not be used in pregnancy nor patients
with hepatic, renal, or CV disease.

You do not have colchicine to treat gout. What would be the 2nd line agent to use?
NSAIDs, though they have actually overtaken colchicine to be first-line agents because of the adverse
diarrheal effects of colchicine.

If colchicine does become available, when would be the most appropriate time to initiate
colchicine therapy in this patient?
Answer choices dealt with administering alongside the NSAID, or withdrawing the NSAID and waiting
for its levels to decrease before administering colchicine, but colchicine needs to be administered
within 24-48 hours of onset of attack to be effective.

What is the method of action of tolbutamide?


It is a sulfonylurea that stimulates insulin release from ȕ cells by binding to the SUR1 subunit and
blocking the ATP-sensitive K+ channel. This depolarizes the cell and allows calcium entry to induce
insulin release from vesicles.

How do acyclovir and AZT function similarly?


Acyclovir is a purine/pyrimidine analog that gets phosphorylated three times, first by the viral enzyme
thymidine kinase, and other two times by host cell enzymes. Acyclovir then competes with dGTP nad
gets incoporated into DNA, causing chain termination and inhibiting viral DNA polymerase. AZT,
otherwise known as Zidovudine (ZDV), is also a nucleoside/nucleotide inhibitor, of reverse
transcriptase. It gets phosphorylated by cellular enzymes and incorporated into viral DNA by reverse
transcriptase, terminating DNA because of its lack of 3’’OH.

Which of the following drugs is used to treat asymptomatic amebiasis?


Diloxanide furoate.

Which drug would you use to treat strains of Plasmodium falciparum found to be resistant to
chloroquine?
I chose mefloquine, because I remember that is used for prophylaxis for chloroqine-resistant
falciparum. But other recommended drugs are atovaquone-proguanil, artemether-lumefantrine, and
quinine + doxycycline, which I think were answer choices too.

Which of the following vitamins are administered to babies immediately after birth?
Vitamin K

Which of these antifungal drugs works by blocking a step in ergosterol synthesis?


Terbinafine. It is an allylamine that inhibits squalene epoxidase which normally would convert
squalene to squalene-2,3 oxide. I got this wrong because I thought griseofulvin does this, but it is the
drug that disrupts mitotic spindle and inhibits mitosis, whereas capsofungin, an echinocandin, inhibits
synthesis of ȕ(1-3)-D-glucans in the fungal cell wall.

A patient came in with acute chest pain which worsened upon exertion. He was given ȕ-blockers
to treat this angina, but then developed shortness of breath. Which ȕ-blocker was most likely
given to him?
Propranolol, because this is a nonselective ȕ1 and ȕ2 antagonist that causes bronchoconstriction as an
adverse effect.

The above patient was switched to another drug to treat his angina by inducing vasodilation
without causing shortness of breath. Which drug was given to him?
I said hydralazine because it is a vasodilator, but apparently it is not used to treat angina, so I don’’t
know.

Which of the following pairings is correct with respect to antibiotic and what it treats?
I got this wrong; I said streptomycin is used to treat anaerobes, but got it mixed up. Streptomycin is an
aminoglycoside and is actually transported via an O2-dependent mechanism, thus it is used to treat
Gram-negative aerobes.

After treatment for an infection, a patient is found to have vacuolated RBCs on peripheral blood
smear. Which antibiotic was most likely given?
Chloramphenicol. According to the CRC Desk Reference for Pharmacology, chloramphenicol causes
both dose-dependent and dose-independent hematologic reactions. Fatal aplastic anemia occurs in
genetically susceptible patients taking chloramphenicol on a long-term basis. Reversible and dose-
dependent disturbances of hemopoiesis can also arise, characterized by altered maturation of red blood
cells, vacuolated nucleated red blood cells in the marrow, and reticulocytopenia (in addition to grey
baby syndrome). Not minocycline, as that is a tetracycline, which has most prominent side effects as
discoloration and hypoplasia of teeth, and stunting of growth in children and fetuses. There is dizziness
and vertigo especially with minocycline.

Which of the following statements is true with regards to the metabolism of alcohol?
Alcohol is absorbed throughout the entire length of the GIT at zero-order kinetics. I don’’t know if this
is true or not, but metabolism via alcohol dehydrogenase occurs at zero-order kinetics.

A female patient suffering from menorrhagia complains of generalized weakness. She is found to
be pale and her hemoglobin count is low. What would you use to treat this patient?
Oral agents of iron supplement should be used first, so ferrous sulfate. Only if the patient is unable to
tolerate oral iron should IV iron dextran be used because of the risk of hypersensitivity reaction with
iron dextran.

Local anesthetics work by blocking action potentials traveling through which kind of nerve
fibers?
Local anesthetics block smaller fibers before larger ones. Thus the small, lightly myelinated B fibers
(preganglionic autonomic), the small unmyelinated C fibers (mediating pain sensation), and the small
myelinated Aį fibers (mediating pain and temperature sensations) are blocked before larger myelinated
AȖ, Aȕ, and AĮ fibers (carrying postural touch, pressure, and motor information).

Lidocaine consists of an aromatic ring and a tertiary amine on either side connected by what
chemical group?
Amide.

A patient who has overdosed on a drug is treated by the physician by making his urine acidic to
excrete more of the drug. What kind of drug would be excreted the fastest in acidic urine?
Henderson-Hasselbach equation: pH –– pKa = log [unprotonated]/[protonated]
Highly basic, with pKa of 7.0. Basic drugs are excreted faster in acidic urine because they become
protonated and not lipophilic so are unable to be reabsorbed in the renal tubule. Acidic drugs are
excreted faster in basic urine.

A patient comes in with an ulcerative red lesion on the face which appeared after exposure to sun.
What does the patient have?
Actinic keratosis, which is a precursor to squamous cell carcinoma of the skin.

NBME
A 56 year old man with a long history of smoking and chronic bronchitis presents with fever and
coughing up green-colored sputum. Culture shows Gram positive lancet-shaped diplococci.
What kind of vaccine could have prevented this illness from occurring?
This is an infection with Streptococcus pneumoniae, which can be prevented with a polysaccharide
vaccine, not a recombinant vaccine.

A boy with nuchal rigidity and photophobia has a sample taken from CSF as well as pharyngeal
swab. Culture from the pharyngeal swab shows Gram negative diplococci with pili formation
whereas culture from the CSF sample shows the same organism without pili. What is responsible
for this difference?
This is Neisseria meningitides, which undergoes phase variation on switch protein expression on and
off.

A young boy presents with headache and neck stiffness along with a petechial rash all over the
body. Cultures bring up Gram negative diplococci. How do these microbes evade destruction by
the host’’s immune system?
Escape phagosome

When strains of influenza virus undergo changes, they may cause pandemics. What kind of
genetic changes are responsible for causing pandemics?
Reassortment of viral genome responsible for genetic shift, whereas random mutations cause genetic
drift and epidemics. I couldn’’t decide between reassortment or recombination, and ended up choosing
the wrong answer.

A male patient who is seeing you for major depressive disorder states ““I can’’t stand my boss
anymore, I think I just want to kill him with my shotgun.”” The best step next for you to take
would be:
Call the police and warn the boss about a possible death threat. Call the family and tell them that he is
psychotic. Prescribe higher doses of antidepressants. Advise him to go through a course of anger
management therapy.

A patient is undergoing treatment for non-small cell lug cancer with paclitaxel. What is the
mechanism of action of paclitaxel?
Paclitaxel along with docetaxel are taxanes that promote and overstabilize microtubules so that they are
nonfunctional, and the cell undergoes apoptosis

A male patient complains of pain in the right thigh, especially when walking. CT scans show a
tumor infiltrating the anterolateral thigh. Which muscle is this tumor affecting the most?
Gracilis, rectus femoris, or vastus intermedius?
A female worker who climbs stairs often injures her knee causing compartmentalization of the
deep fibular nerve where it travels in a groove. Damage to this nerve would cause the most
profound effect on which function of the lower limb?
Ankle dorsiflexion or foot inversion or foot eversion?

An experimental study is done to see how effective Coenzyme Q10 is in treating a disease. A coin
is flipped to determine which patients will be treated with standard therapy and which ones with
standard therapy plus Coenzyme Q10. What kind of study is this?
Randomized clinical trial- this sees whether treatment has an effect on outcome of disease. All patients
in study have the disease whereas a case-control study is observational and retrospective, comparing a
group of people with disease to a group without and seeing what happened. A cohort study is
observational and prospective. It compares a group with a given risk factor to a group without to assess
whether the risk factor increases likelihood of disease.

The following are results of a case-control study. Calculate the odds ratio of exposure and risk of
disease.
Disease
Yes No
Case Control
Exposure Yes 250 (a) 50 (b)
No 250 (c) 150 (d)
Odds ratio is the odds of exposure if case divided by odds of exposure if control.
Odds of exposure if case = [a/(a+c)]/[c/(a+c)] = a/c
Odds of exposure if control = [b/(b+d)]/[d/(d+b)] = b/d
Odds ratio = (a/c)/(b/d) = (ad)/(bc) = (250*150)/(50*250) = 3

A study is done to test a new drug. It is found to have 40% chance (CI 0.2-0.89) of reducing
morbidity whereas control drug has 30% chance. It also has a 70% chance (CI 0.4-1.11) of
reducing mortality whereas the control has 40% chance. What can you say about these results?
The confidence interval of the new drug overlaps with the values of the control, so I think neither
findings are statistically significant.

A weight lifter feels pain in the right lower quadrant of the abdomen. Shown is a resection of the
intestine that looks like a red infarct. What is the most likely cause?
I wasn’’t sure between embolism, strangulation, volvulus, intussusception, because I think all could
cause red infarcts of the intestines. Went with strangulation because of the history of weight-lifting,
which causes increased intra-abdominal pressure and possible inguinal hernia in the right lower
quadrant, which would then lead to strangulation.

A mother brings her child to you because of a lesion on the face as shown:
What would you tell the anxious parent?
You have to first correctly identify this as a hemangioma, so not to worry as it is benign and will most
likely regress within a few years.

A pregnant female comes in because of vaginal spotting. She is 6 weeks pregnant based on
calculations from her last menstrual period, but her uterus size is that of being 14 weeks
pregnant. Palpation reveals bilateral adnexal masses. ȕHCG levels are much higher than
normal. What is the diagnosis?
I wasn’’t sure because the whole history points towards hydatidiform mole except for the bilateral
adnexal masses, which I don’’t think happens in hydatidiform mole; ectopic pregnancy was another
answer choice, which would cause adnexal masses, but bilateral ectopic dizygotic twins?

An elderly man on chronic treatment of NSAIDS for osteoarthritis comes in with abdominal
pain. Shown is a histological biopsy of his colon. What is your diagnosis?
I said hemochromatosis because I thought I saw iron deposits in the lining of the colon, though the
classic locations of iron deposits are the liver and pancreas.

Studies are being done on a polymorphism which causes an increased expression of the cytokine
IL-13. IL-13 binds to the IL-4 receptor. What effect would you expect to see in a patient with
this polymorphism?
IL-4 is required for isotype switching to IgE, so increased IL-13 would lead to increased secretion of
IL-13.

A young boy presents with fever, pharyngitis, and lymphadenopathy. Heterophil antibodies are
detected in the patient’’s serum. The etiological agent can be found latent in which cells a year
later?
You have to recognize this as an infection by EBV, which is a part of the subfamily
Gammaherpesvirinae along with HHV-8. So it establishes latency in B cells (lymphoid tissue), not
hepatocyte. Alphaherpesvirinae includes HHV 1, 2 & 3, and establishes latency in neurons.
Betaherpesvirinae includes HHV 5, 6 & 7, and establishes latency in glands (kidneys, liver).

A patient with defective lysyl hydroxylase would have difficulty in which step of collagen
synthesis?
Forming the amino acid hydroxyproline. I got this wrong because I confused lysyl hydroxylase and
lysyl oxidase. First collagen Į chains are synthesized in the RER, then hydroxylation occurs in the ER
as well as glycosylation which forms procollagen (triple helix of 3 collagen Į chains). This
procollagen is exocytosed and cleavage of terminal regions yields tropocollagen. Cross-linkingof
many tropocollagens together by lysyl oxidase is what makes the end product collagen fibrils.
A young girl comes in with hypermobile joints, hyperextensible skin, and easy bruising. Her
grandmother had a similar disorder. Faulty synthesis of what protein is causing the girl’’s
symptoms?
I wasn’’t sure if this was Marfan’’s or Ehlers-Danlos syndrome. If Marfan’’s, the answer would be
fibrillin. She has Ehlers-Danlos syndrome, which is due to faulty collagen synthesis, not elastin.

A music teacher is brought in by family because of 8 month duration of aberrant behavior. She is
described as her friends as being prudish and refined and respected by her students. But
recently she has become rude and curses at her students. She has also become quite promiscuous
and has had several sexual encounters with men she had just met. She most likely has a lesion in
the brain in which area?
Posterior cingulate gyrus, or anterior cingulated gyrus?

A 56 year old lady comes in because she has felt quite anxious recently. She normally goes to
church but decided to stay at home this week. She has begun to skip her job as a bookkeeper.
She is postmenopausal but keeps symptoms under control with hormone replacement therapy.
She is taking medication for 15 year history of general anxiety disorder. What is your diagnosis?
I have no idea. Answers were menopause, GAD, MDD, estrogen toxicity, and diazepam toxicity.

If there were an occlusion in the artery indicated in the image taken from a basilar arterial tree
angiography, what symptoms would you expect to see?
I think the image was pointing to the PCA, which if occluded would cause contralateral visual field
defects. I got confused by the wording because I thought contralateral visual field defect meant
blindness in the contralateral eye and normal vision in the ipsilateral eye, which I know does not
happen because occlusion of the PCA affects the ipsilateral visual cortex which is responsible for visual
fields from both eyes. That would be contralateral homonymous hemianopia, which does affect both
eyes, but the contralateral is just describing the visual field defect. (left side or right side of visual field,
not left eye or right eye).

A woman comes in because she is unable to move her left eye to the right when she abducts her
right eye. Her right eye begins having saccadic movements when she tries this. Convergence is
normal. She most likely has a lesion where?
Left medial longitudinal fasciculus.

A lady presents to the emergency room with vaginal bleeding. She is accompanied by her 15 year
old son because she cannot speak English, and he helps to translate for her. The attending
physician wants to conduct a physical examination on her, but only knows a few words in the
patient’’s native language. What would be the appropriate next step to do?
Call for a certified interpreter. Not ask your colleague to help translate because he speaks the patient’’s
native language, nor ask the son to step behind the curtain with you to continue interpreting.

An HIV-positive patient comes in with headache and confusion. Despite treatment, he passes
away. An autopsy was done, and he was found to have jelly-like meninges. A biopsy was taken,
showing the causative microorganism. What caused this man’’s disease?
Cryptococcus neoformans

A man comes in with a pruritic erythematous rash along the webbings of his fingers, at his wrists,
axillae, and genitals. He is an avid hiker and stay with his family in a cabin near the woods. The
whole family has similar rash. What do you do next for the patient?
You have to recognize that the patient and family has scabies infestation and so you prescribe
permethrin. Not advise to wear long sleeves when he goes hiking, spray DEET to prevent insect bites,
nor have an exterminator disinfect the whole house.

A male smoker is on bupropion to aid in smoking cessation, but he states that it is not working.
How would you check to make sure if he is keeping with his medication?
I don’’t know, I said to check cathinone levels.

A young boy with recent history of UTI presents to the hospital with degrading condition. He is
found to have hydroureter and hydronephrosis.

What would you expect to find on a histopathological slide of this specimen?


Interstitial nephritis?

A male patient is on ciprofloxacin to treat a UTI. He has a medication history of X, Y, Z, and


aluminum hydroxide to treat heartburn. Which of these will decrease the effectiveness of
ciprofloxacin?
Don’’t remember the other drugs listed, but the answer is AlOH because the divalent cations in antacids
interfere with the absorption of fluoroquinolones. Remember also that elimination is via renal tubular
secretion, so probenecid would block this and make the drug levels too high.

Once HIV penetrates a cell, it can undergo rapid mutation. This happens because it lacks what?
Its reverse transcriptase lacks 3’’Æ5’’ exonuclease activity.

A man comes in with history of multiple infections. He has high HIV viral loads, but an ELISA
test comes up negative. What went wrong?
I don’’t know. He has been infected with HIV-2?

A diagram is shown of a sympathetic neuron synapse with adrenergic receptors and a


parasympathetic neuron synapse with cholinergic receptors, and various receptors and enzymes
labeled. A drug that blocks which one would increase urination in a patient who has come in
with increased urinary difficulty and frequency?
I remembered that Į1 receptors on genitourinary smooth muscle cause constriction, so blocking these
would improve urine flow. Expectedly, Į1 blockers are drugs of choice for symptomatic relief of BPH.
I had to choose between that and blocking cholinesterase, because I thought this may help to improve
symptoms too. Indeed, but anticholinesterases are used more for bladder atony (and myasthenia gravis)
than for BPH.

A 56 year old lady comes in with a broken femur. Which factor will affect her rate of
rehabilitation the most?
I said activity level before the fracture.

A young sexually active female presents with a painless ulcer on the labia. How do you test
whether or not this sexually-transmitted infection is syphilis?
Enzyme immunoassay to see if the patient has anti-treponemal antibodies, not if she has treponemal
antigens in her blood.

A 50 year old lady presents with nausea and hot flushes. She states that she is not
postmenopausal. If she has menopause, what levels would you expect FSH and LH to be relative
to normal?
ĹĹFSH, ĹLH. ĹĹFSH is confirmatory test.

A man comes in with lethargy and confusion. He states that he has not found the need to urinate
anymore. Lab findings show that his serum sodium levels are low. What can you say about his
urine sodium and urine osmolality?
Both will be high.

A couple goes to fertility consulting because they have not been successful in getting her
pregnant. The husband was found previously to be normal. The wife is anovulatory, and a
biopsy of the wife’’s uterus reveals endometrium with prominent, developed secretory glands in
the epithelium. What is the cause of this finding?
I don’’t know, I thought that she had dysfunctional uterine bleeding, so I said increased secretion of
estradiol.
A young boy is brought in because of mental retardation. His urine is found to have increased
levels of 3 branched amino acids. What other characteristic would you expect to find in the
urine?
Maple syrup odor.

A man woke up one morning with a tender, red ankle. Serum levels of uric acid are found to be
high. If this condition persists, what is the man at risk of developing?
The man has gout, so osteoarthritis?

A woman with family history of Huntington’’s disease can get DNA testing to check to see if she
has inherited the gene that will cause onset of the disease later in life. The benefit for getting
DNA testing done is to:
I said begin prophylaxis for dementia, but there is no medical intervention that can slow or prevent
onset of the disease. The main benefit of ““presymptomatic”” DNA testing is psychological, to relieve
any anxiety or uncertainty the patients may have, as those found either to be high risk or low risk were
also found to have less rates of depression whereas those still with uncertainty had greater rates of
depression.

How do you tell monoclonality of cancer cells in endometrial carcinoma?


I don’’t know, I said check the HLA subtype, but I think the answer may be check X-linked protein
expression because of X-inactivation. Monoclonal cells would have the same X chromosome
inactivated. Other answers were metalloproteinase expression, rate of metastasis, laminin receptor
density.

A woman with Hodgkin’’s disease and a history of fever and night sweats is found to have several
metastases in the hilar lymph nodes. What stage of Hodgkin’’s disease is this?
Stage IB

All stages are further divided on the basis of the absence (A) or presence (B) of the following systemic
symptoms: significant fever, night sweats, unexplained loss of more than 10% of normal body weight.

An overweight man with hyperlipidemia is being treated with lovastatin, but his LDL levels still
remain high. Which drug would you add to his regimen that binds to cholesterol in the intestine
and prevent absorption?
Ezetimibe. Statins and ezetimibe work hand in hand because statins inhibit cholesterol production and
thus increase intestinal cholesterol absorption, whereas ezetimibe inhibits cholesterol absorption and
thus increases cholesterol production. So both are commonly given simultaneously to complement
each other’’s effects.

A young boy with headache and fever is given aspirin as treatment. He develops confusion and
mental agitation. Which drug should you switch to to treat the boy’’s fever?
The boy has Reye’’s syndrome, so acetaminophen is drug of choice. Ibuprofen is safe too.
A patient with Parkinson’’s disease comes to you because he is on levodopa medication but still
has symptoms of rigidity and tremor. What drug can you prescribe to him that will prevent
levodopa metabolism in the brain?
I only remembered carbidopa which inhibits dopa decarboxylase, but I knew that it only works in the
periphery, so didn’’t know the answer. Forgot that inhibition of MAO and COMT in the CNS is another
way to prevent breakdown of carbidopa. Deprenyl (selegiline) and rasagiline inhibit MAO, whereas
tolcapone inhibits COMT in the brain.

Potrebbero piacerti anche